You are on page 1of 83

Trade Name Group Indication Side effect Precaution Interaction Adverse effects/ toxic Nursing care/ patient education

(Generic Name) reactions


Triact Antacids -acid indigestion nausea, tell your doctor or pharmacist phosphate Severe headache and passing out or trouble
constipation, if you are allergic to supplements (e.g., bleeding nose breathing
-used to treat the symptoms of too diarrhea, or aluminum hydroxide; or to potassium
much stomach acid such as stomach headache magnesium; or to simethicone phosphate), sodium
upset, heartburn, and acid polystyrene
indigestion frequent alcohol use, severe sulfonate
loss of body water
-relieve symptoms of extra gas such (dehydration /fluid digoxin, iron,
as belching, bloating, and feelings of restriction), kidney problems pazopanib,
pressure/discomfort in the (including kidney stones), tetracycline
stomach/gut. diabetes antibiotics,
quinolone
restrict your intake of antibiotics such as
aspartame (or phenylalanine) ciprofloxacin

Pregnancy and breast-feeding


Trade Name Group Indication Side effect Precaution Interaction Adverse effects/ Nursing care/ patient education
(Generic Name) toxic reactions
Mylanta Antacids, treat the symptoms of too nausea, constipation, Caution: frequent digoxin, iron, Muscle weakness patient education
Antireflux Agents much stomach acid such diarrhea, or headache. alcohol use, pazopanib, Black stools Do not use extra dose to make up
& Antiulcerants as stomach dehydration/fluid tetracycline for a missed dose.
upset, heartburn, and Magnesium cause diarrhea. restriction, kidney antibiotics,
acid indigestion Aluminum cause problems quinolone
constipation. Drink plenty of antibiotic like
fluids and exercise. restrict your intake ciprofloxacin
of aspartame or
Using an antacid that phenylalanine
contains only aluminum can
help control diarrhea. Pregnancy and
breastfeeding
Trade Name Group Indication Side effect Precaution Interaction Adverse effects/ Nursing care/ patient education
(Generic toxic reactions
Name)
Pepcidine Antacids, -active duodenal ulcer. Occasional : contraindication: DRUG: Antacids slightly Agranulocytosis, BASELINE ASSESSMENT
(Famotidine) Antireflux -Prevention, maintenance of Headache. Hypersensitivity to decrease the bioavailability pancytopenia, Assess epigastric/abdominal pain.
Agents & duodenal ulcer recurrence. other H2 antagonists. of famotidine. May reduce thrombocytopenia
Antiulcerants -active benign gastric ulcer, Rare : serum concentration of occurs rarely INTERVENTION/EVALUATION
-pathologic GI hypersecretory Confusion, Cautions: ketoconazole and Monitor daily pattern of bowel
conditions. constipation, Renal/hepatic itraconazole. May decrease activity, stool consistency. Monitor
-GERD. diarrhea, impairment, elderly, absorption of atazanavir, for diarrhea, constipation,
-heartburn, dizziness thrombocytopenia. itraconazole, ketoconazole. headache. Assess for confusion in
-acid indigestion, sour stomach. elderly.
FOOD: Alcohol may cause
OFF LABEL: gastric mucosal irritation. PATIENT/FAMILY TEACHING
H. pylori eradication, risk • May take without regard to
reduction of duodenal ulcer LAB VALUES: Interferes meals, antacids. • Report
recurrence (part of multidrug with skin tests using headache. • Avoid excessive
regimen), stress ulcer allergen extracts. May amounts of coffee, aspirin. •
prophylaxis in critically ill pts, increase serum alkaline Report persistent symptoms of
relief of gastritis. phosphatase, ALT, AST. heartburn, acid indigestion, sour
May decrease platelet stomach.
count.
Trade Name Group Indication Side effect Precaution Interaction Adverse effects/ Nursing care/ patient education
(Generic Name) toxic reactions
Pantoloc Antacids, Duodenal, gastric ulcer, Esophageal Rare: Contraindications: DRUG: May Hyperglycemia BASELINE ASSESSMENT
(Pantoprazole) Antireflux Agents reflux, esophagitis, GERD, Peptic ulcer Diarrhea, Hypersensitivity increase effects of occurs rarely. Question history of GI disease,
& Antiulcerants disease, Zollinger-Ellison syndrome headache, to proton pump warfarin. May ulcers, GERD.
dizziness, inhibitors (e.g., decrease effects of
PO: Treatment, maintenance of healing pruritus, rash. omeprazole). clopidogrel, INTERVENTION/EVALUATION
of erosive esophagitis associated with atazanavir. Evaluate for therapeutic response
GERD. Reduction of relapse rate of Cautions: May (relief of GI symptoms). Question
heartburn symptoms in GERD. increase risk of if GI discomfort, nausea occur.
LAB VALUES:
Treatment of hypersecretory conditions fractures, GI
May increase
including Zollinger Ellison syndrome. infections PATIENT/FAMILY TEACHING
serum creatinine,
• Report headache, onset of black,
cholesterol, uric
IV: Short-term treatment of erosive tarry stools, diarrhea. • Avoid
acid, glucose,
esophagitis associated with alcohol. • Swallow tablets whole;
lipoprotein, ALT.
GERD, treatment of hypersecretory do not chew, crush, dissolve, or
conditions. divide. • Best if given before
breakfast. May give without regard
OFF-LABEL: Peptic ulcer disease, to food.
active ulcer bleeding (injection), adjunct
in treatment of H. pylori, stress ulcer
prophylaxis in critically ill pts.
Trade Name Group Indication Side effect Precaution Interaction Adverse effects/ Nursing care/ patient education
(Generic toxic reactions
Name)
Dulcolax (PR) contact Treatment of Frequent: Some degree Contraindications: DRUG: Antacids electrolyte or BASELINE ASSESSMENT
(Bisacodyl) laxatives constipation, of abdominal Abdominal pain, may decrease effect, metabolic Observe for evidence of constipation. Assess pattern of
colonic discomfort, nausea, mild appendicitis, cause premature disturbances bowel activity, stool consistency.
evacuation cramps, intestinal dissolution of enteric (hypokalemia,
before faintness. obstruction, nausea, coating and possible hypocalcemia, INTERVENTION/EVALUATION
examinations undiagnosed rectal gastric irritation. metabolic Encourage adequate fluid intake. Assess bowel sounds for
or Occasional: Rectal bleeding, vomiting, acidosis, peristalsis. Monitor daily pattern of bowel activity, stool
procedures. administration: burning pregnancy, FOOD: Milk may alkalosis), consistency; record time of evacuation. Assess for
of rectal mucosa, mild lactation. cause rapid persistent abdominal disturbances. Monitor serum electrolytes in
proctitis. dissolution of diarrhea, those exposed to prolonged, frequent, or excessive use of
Cautions: Long- bisacodyl. vomiting, medication.
Long-term use may term use may lead to muscle
result in laxative laxative LAB VALUES: May weakness, TEACHING
dependence, chronic dependence, loss of increase serum malabsorption, increase fluid intake, exercise, high fiber diet. Do not take
constipation, loss of normal bowel glucose weight loss. antacids, milk, or other medication within 1 hr of taking
normal bowel function function concentration. May medication (decreased effectiveness). Report unrelieved
decrease serum constipation, rectal bleeding, muscle pain or cramps,
potassium (due to dizziness, weakness.
fluid loss).
Trade Name Group Indication Side effect Precaution Interaction Adverse effects/ Nursing care/ patient education
(Generic Name) toxic reactions
(Lactulose) Laxatives, Prevention, Occasional: Contraindications: DRUG: None Severe diarrhea Encourage adequate fluid intake. Assess bowel
Purgatives treatment of portal- Abdominal cramping, Pts requiring a low significant. indicates sounds for peristalsis. Monitor daily
systemic flatulence, increased galactose diet. overdose. Long- pattern of bowel activity, stool consistency; record
encephalopathy* thirst, abdominal HERBAL: None term use may time of evacuation. Assess for abdominal
(including hepatic discomfort. Cautions: Diabetes significant. result in laxative disturbances. Monitor serum electrolytes in pts with
precoma, coma); mellitus, hepatic dependence, prolonged, frequent,
treatment of Rare: Nausea, impairment, chronic excessive use of medication.
LAB VALUES:
constipation. vomiting dehydration. constipation, loss
May decrease
of normal bowel Teaching:
serum potassium
*is a function. Evacuation occurs in 24–48 hrs of initial dose.
(GI loss).
neuropsychiatric Institute measures to promote defecation: increase
disorder that occurs fluid intake, exercise, high-fiber diet.
secondary to
chronic liver
disease.
Trade Name Group Indication Side effect Precaution Interaction Adverse effects/ Nursing care/ patient education
(Generic toxic reactions
Name)
Metamucil bulk-forming Management of chronic Rare: Some degree Contraindications: DRUG: None Esophageal/bowel INTERVENTION/EVALUATION
美達施 laxative constipation, in irritable of abdominal Fecal impaction, GI significant. obstruction may Encourage adequate fluid intake. Assess bowel
(psyllium) bowel syndrome, as discomfort, nausea, obstruction, occur if sounds for peristalsis. Monitor daily pattern of
adjunctive therapy in mild abdominal undiagnosed LAB VALUES: administered with bowel activity, stool consistency. Monitor serum
diverticular disease and cramps, griping, abdominal pain. May increase insufficient liquid electrolytes in pts exposed to prolonged, frequent,
constipation caused by faintness. serum glucose. (< 250 ml). excessive use of medication.
antacid administration in Cautions: Esophageal May decrease
duodenal ulcer; in the strictures, ulcers, serum PATIENT/FAMILY TEACHING
bowel management of stenosis, intestinal potassium. • Take each dose with full glass (250 ml) of
patients with haemorrhoids; adhesions, difficulty water. • Inadequate fluid intake may cause GI
constipation during swallowing, obstruction. • Institute measures to promote
pregnancy, convalescence management of defecation (increase fluid intake, exercise, high-
and senility. irritable bowel fiber diet). • consult a doctor if sudden change in
syndrome (IBS), bowel habits persisting for 2 weeks, abdominal
OFF-LABEL: Diarrhea, elderly. pain, nausea, vomiting, chest pain, difficulty in
chronic constipation, swallowing or breathing, or rectal bleeding
inflammatory bowel occurs, or if constipation persists >7 days.
disease.
Trade Name Group Indication Side effect Precaution Interaction Adverse effects/ toxic Nursing care/ patient education
(Generic Name) reactions
Senokot Laxatives, Short-term use Frequent: Red, brown Contraindications: DRUG: May Prolonged INTERVENTION/EVALUATION
(Senna) Purgatives for discoloration of urine. Undiagnosed acute or decrease transit time use/overdose may Encourage adequate fluid intake. Assess
constipation , to Occasional: Some persistent abdominal of concurrently result in electrolyte, bowel sounds for peristalsis. Monitor daily
evacuate colon degree of abdominal symptoms, administered oral metabolic pattern of bowel activity, stool consistency.
:
before discomfort, nausea, appendicitis, intestinal medications, disturbances (e.g., Assess for GI disturbances. Monitor serum
bowel/rectal mild cramping, obstruction or decreasing hypokalemia, electrolytes in pts exposed to prolonged,
examinations. faintness. perforation, nausea, absorption. hypocalcemia, frequent, excessive use of medication.
vomiting. metabolic acidosis or
Long-term use may LAB VALUES: May alkalosis), vomiting, PATIENT/FAMILY TEACHING
result in laxative Cautions: Prolonged increase serum muscle weakness, • Urine may turn red or brown (only
dependence, chronic use (longer than 1 wk) glucose. May persistent diarrhea, temporary and not harmful). • Institute
constipation, loss of may lead to decrease serum malabsorption, measures to promote defecation (increase fluid
normal bowel dependency, fluid and potassium. weight loss. intake, exercise, high-fiber diet). • Laxative
function. electrolyte imbalance, effect generally occurs in 6–12 hrs but may
vitamin and mineral take 24 hrs. • Do not take other oral
deficiency medication within 1 hr of taking senna
(decreased effectiveness).
Cardiovascular system
Trade Name Group Indication Side effect Precaution Interaction Adverse effects/ toxic reactions Nursing care/ patient education
(Generic Name)
Lanoxin Cardiac Treatment of mild Dizziness, Contraindications: DRUG: Amiodarone may GI disturbances (anorexia, BASELINE ASSESSMENT
(Digoxin) Drugs, cardiac to moderate HF, headache, Ventricular fibrillation. increase nausea, vomiting), neurologic Assess apical pulse. If pulse is 60
glycosides atrial fibrillation diarrhea, concentration/toxicity. Beta- abnormalities (fatigue, or less/ min (70 or less/min for
(rate-controlled). rash, visual blockers, calcium channel headache, depression, children), withhold drug. Blood
disturbances. Cautions: Renal blockers may have additive weakness, drowsiness, samples are best taken 6–8 hrs
impairment, sinus effect on slowing AV nodal confusion, nightmares). Facial after dose or just before next dose.
OFF-LABEL: nodal disease, acute MI conduction. Potassium- pain, personality change, ocular
Fetal tachycardia (within 6 mos), second- depleting diuretics may disturbances (photophobia,
with or without or third-degree heart increase toxicity due to light flashes, halos around INTERVENTION/EVALUATION
hydrops; decrease block (unless hypokalemia. bright objects, yellow or green Monitor pulse for bradycardia,
ventricular rate in functioning Sympathomimetics may color perception) may occur. ECG for arrhythmias for 1–2 hrs
supraventricular pacemaker), concurrent increase risk of arrhythmias. after administration (excessive
tachyarrhythmias. use of strong inducers slowing of pulse may be first
or inhibitors of P- Sinus bradycardia, AV block, clinical sign of toxicity). Assess
glycoprotein HERBAL: Ephedra may ventricular arrhythmias noted. for GI disturbances, neurologic
(cyclosporine), increase risk of arrhythmias. abnormalities (signs of toxicity)
hyperthyroidism, Licorice may cause sodium q2–4h during loading dose (daily
hypothyroidism, and water retention, loss of during maintenance). Monitor
potassium. serum potassium, magnesium,
hypokalemia, calcium, renal function.
hypocalcemia. Therapeutic serum level: 0.8–2
FOOD: Meals with increased ng/ml; toxic serum level: greater
fiber (bran) or high in pectin than 2 ng/ml.
may decrease absorption.
PATIENT/FAMILY TEACHING
• Follow-up visits, blood tests
are an important part of therapy. •
Follow guidelines to take apical
pulse and report pulse 60 or
less/min (or as indicated by
physician). • Wear/carry
identification of digoxin therapy
and inform dentist, other physician
of taking digoxin. • Do not
increase or skip doses. • Do not
take OTC medications without
consulting physician. • Report
decreased appetite,
nausea/vomiting, diarrhea, visual
changes
Trade Name Group Indication Side effect Precaution Interaction Adverse effects/ toxic reactions Nursing care/ patient education
(Generic Name)
Lasix Diuretics Treatment of Expected: Increased contraindications: DRUG: Amphotericin B, water loss/electrolyte depletion, BASELINE ASSESSMENT
(furosemide) edema associated urinary frequency/ Anuria. nephrotoxic, ototoxic resulting in hypokalemia, Check vital signs, esp. B/P, pulse, for
with HF and volume. Frequent: medications may increase hyponatremia, dehydration. hypotension before administration.
renal/hepatic Nausea, dyspepsia, Cautions: Hepatic risk of nephrotoxicity, thrombosis, circulatory collapse, Assess baseline serum electrolytes,
disease; acute abdominal cramps, cirrhosis, hepatic ototoxicity. May increase sudden death. Acute esp. for hypokalemia. Assess skin
pulmonary diarrhea or coma, severe risk of lithium toxicity. hypotensive episodes may turgor, mucous membranes for
edema. Treatment constipation, electrolyte Other medications causing occur, sometimes several days hydration status; observe for edema.
of hypertension, electrolyte depletion, hypokalemia may increase after beginning therapy. Assess muscle strength, mental
either alone or in disturbances. prediabetes, risk of hypokalemia. status. Note skin temperature,
combination with diabetes, systemic moisture. Obtain baseline weight.
other Occasional: Dizziness, lupus Ototoxicity (deafness, vertigo, Initiate I&O monitoring.
antihypertensives. light-headedness, erythematosus. Pts HERBAL: Ephedra, tinnitus) may occur, esp. in pts
headache, blurred with prostatic ginseng, yohimbe may with severe renal impairment.
vision, paresthesia, hyperplasia/urinary worsen hypertension. INTERVENTION/EVALUATION
photosensitivity, rash, stricture. Garlic may increase Monitor B/P, vital signs, serum
fatigue, bladder antihypertensive effect. Can exacerbate diabetes electrolytes, I&O, weight. Note
spasm, restlessness, mellitus, systemic lupus extent of diuresis. Watch for
diaphoresis. erythematosus, gout, symptoms of electrolyte imbalance:
Hypokalemia may result in changes
Rare: Flank pain. LAB VALUES: May pancreatitis. Blood dyscrasias in muscle strength, tremor, muscle
increase serum glucose, have been reported. cramps, altered mental status, cardiac
BUN, uric acid. May arrhythmias; hyponatremia may
decrease serum calcium, result in confusion, thirst,
chloride, magnesium, cold/clammy skin.
potassium, sodium.
TEACHING
• Expect increased frequency, volume
of
urination. • Report palpitations, signs
of
electrolyte imbalances (noted
previously),
hearing abnormalities (sense of
fullness in
ears, tinnitus). • Eat foods high in
potassium
such as whole grains (cereals),
legumes,
meat, bananas, apricots, orange juice,
potatoes (white, sweet), raisins.
• Avoid sunlight, sunlamps.
Trade Name Group Indication Side effect Precaution Interaction Adverse effects/ Nursing care/ patient education
(Generic Name) toxic reactions
Cordarone antianginal Management of Expected: Corneal Contraindications: ncrease thioridazine alveolitis, BASELINE ASSESSMENT
(amiodarone) and class III life-threatening microdeposits noted Bradycardia-induced concentration and pulmonary Obtain baseline ALT, AST, alkaline
antiarrhythmic recurrent in almost all pts syncope (except in the produce additive fibrosis, phosphatase, EKG; pulmonary function tests,
drug, cardiac ventricular treated for more than presence of a prolongation of QT pneumonitis, CXR in pts with pulmonary disease. Assess
drug fibrillation, 6 mos (can lead to pacemaker), second- and interval. May acute respiratory B/P, apical pulse immediately before drug is
hemodynamically blurry vision). third-degree AV block increase cardiac distress administered
unstable (except in presence of a effects with other syndrome) may INTERVENTION/EVALUATION
ventricular pacemaker), severe sinus antiarrhythmics. May begin with Monitor for symptoms of pulmonary toxicity
tachycardia (VT) Occasional: PO: node dysfunction, increase effect of progressive (progressively worsening dyspnea, cough).
unresponsive to Constipation, cardiogenic shock. beta-blockers, oral dyspnea and Dosage should be discontinued or reduced if
other therapy. headache, decreased Hypersensitivity to anticoagulants (e.g., cough with toxicity occurs. Assess pulse for quality,
appetite, nausea, iodine. warfarin). May crackles, rhythm, bradycardia. Monitor EKG for cardiac
vomiting, increase decreased breath changes. Assess for nausea, fatigue, paresthesia,
OFFLABEL: paresthesia, concentration, sounds, pleurisy, tremor. Monitor for signs of hypothyroidism
Treatment of photosensitivity, Cautions: May prolong toxicity of HF, or (periorbital edema, lethargy, pudgy hands/feet,
atrial fibrillation, muscular QT interval. Thyroid aripiprazole, hepatotoxicity. cool/pale skin, vertigo, night cramps) and
paroxysmal incoordination. disease, electrolyte colchicine, digoxin, May worsen hyperthyroidism (hot/dry skin, bulging eyes
supraventricular Parenteral: imbalance, hepatic phenytoin. May existing [exophthalmos], frequent urination, eyelid
tachycardia Hypotension, nausea, disease, hypotension, left increase risk of sim arrhythmias or edema, weight loss, difficulty breathing).
(SVT); fever, bradycardia. ventricular dysfunction, vastatin toxicity, produce new Monitor serum ALT, AST, alkaline phosphatase
ventricular pulmonary disease. Pts myopathy, arrhythmias. for evidence of hepatic toxicity. Assess skin,
tachyarrhythmias. taking warfarin, surgical rhabdomyolysis cornea for bluish discoloration in pts who have
Rare: PO: Bitter or pts. been on drug therapy longer than 2 mos.
metallic taste, FOOD: Grapefruit Monitor thyroid function test results. If elevated
decreased libido, products may alter hepatic enzymes occur, dosage reduction or
dizziness, facial effect. Avoid use discontinuation is necessary. Monitor for
flushing, blue-gray during therapy. therapeutic serum level (0.5–2.5 mcg/ml). Toxic
coloring of skin serum level not established.
(face, arms, and
neck), blurred vision, PATIENT/FAMILY TEACHING • Protect
bradycardia, against photosensitivity reaction on skin
asymptomatic exposed to sunlight. • Bluish skin
corneal deposits, discoloration gradually disappears when drug is
rash, visual discontinued. • Report shortness of breath,
disturbances, halo cough. • Outpatients should monitor pulse
vision. before taking medication. • Do not abruptly
discontinue medication. • Compliance with
therapy regimen is essential to control
arrhythmias. • Restrict salt, alcohol intake. •
Avoid grapefruit products. • Recommend
ophthalmic exams q6mos. • Report any vision
changes, signs/symptoms of cardiac arrhythmias

Trade Name Group Indication Side effect Precaution Interaction Adverse effects/ Nursing care/ patient education
(Generic Name) toxic reactions
Betaloc Beta- Treatment of Frequent: Contraindications: (MI) DRUG: bradycardia, BASELINE ASSESSMENT
(Metoprolol) Blockers hemodynamically Diminished sexual Severe sinus bradycardia, Diuretics, other hypotension, Assess baseline renal function, LFT.
stable acute myocardial function, MI with heart rate less than antihypertensives bronchospasm. Assess B/P, apical pulse immediately
infarction (AMI), angina drowsiness, 45 beats/min or systolic may increase Abrupt withdrawal before drug administration (if pulse is
pectoris, hypertension. insomnia, unusual B/P less than 100 mm Hg, hypotensive effect. may result in <60 bpm or systolic B/P is <90 mmHg
fatigue/ weakness. moderate to severe HF, hypoglycemia, diaphoresis,
significant 1 degree heart
st
prolong palpitations,
Toprol XL: Treatment of block, 2 or 3 degree heart
nd rd
hypoglycemic headache, Antianginal: Record onset,
angina pectoris, to reduce Occasional: block. effect of insulin, tremulousness, type (sharp, dull, squeezing), radiation,
mortality or Anxiety, diarrhea, oral hypoglycemics. exacerbation of location, intensity, duration of anginal
hospitalizations in pts with constipation, NSAIDs may angina, MI, pain, precipitating factors (exertion,
HF, already receiving ACE nausea, vomiting, (HTN/Angina): Sinus decrease ventricular emotional stress).
inhibitors, diuretics, and/or nasal congestion, bradycardia, 2 - or 3
nd rd
antihypertensive arrhythmias. May
digoxin; hypertension. abdominal degree heart block, effect. precipitate HF, MI INTERVENTION/EVALUATION
discomfort, cardiogenic shock, overt Sympathomimetics, in pts with heart Measure BP near end of dosing interval
dizziness, difficulty HF, sick sinus syndrome xanthines may disease, thyroid (determines whether BP is controlled
OFF-LABEL: Treatment breathing, cold (except with pacemaker), mutually inhibit storm in those with throughout day). Monitor BP for
of ventricular arrhythmias, hands/feet. severe peripheral arterial effects. Potent thyrotoxi cosis, hypotension, respiration for shortness of
migraine prophylaxis, disease, CYP2D6 inhibitors peripheral ischemia breath. Assess pulse for quality, rate,
essential tremor, pheochromocytoma. (e.g., fluoxetine, in those with rhythm. Assess for evidence of HF:
aggressive behavior, Rare: Altered taste, cimetidine) may existing peripheral dyspnea (esp. on exertion, lying down),
prevent reinfarction post dry eyes, increase vascular disease. night cough, peripheral edema, distended
MI, prevent/treat atrial nightmares, (Extended-Release): concentration. Hypoglycemia may neck veins. Monitor I&O (increased
fibrillation/ atrial flutter, paresthesia, allergic Severe bradycardia, 2 or
nd
occur in pts with weight, decreased urinary output may
hypertrophic reaction (rash, 3 degree heart block,
rd
previously indicate HF). Therapeutic response to
cardiomyopathy, pruritus). cardiogenic shock, Digoxin, verapamil, controlled DM hypertension noted in 1–2 wks.
thyrotoxicosis decompensated HF, sick diltiazem may (may mask signs of
sinus syndrome (except increase risk of hypoglycemia). TEACHING
with functioning bradycardia/heart Antidote: Glucagon • Do not abruptly discontinue
pacemaker). block. (see Appendix K medication. • Compliance with therapy
for dosage). regimen is essential to control
hypertension, arrhythmias. • If dose is
Cautions: Arterial missed, take next scheduled dose (do not
obstruction, double dose). • Go from lying to standing
bronchospastic disease slowly. • Report excessive fatigue,
hepatic impairment, dizziness. • Avoid tasks that require
peripheral vascular alertness, motor skills until response to
disease, hyperthyroidism, drug is established. • Do not use nasal
diabetes mellitus, decongestants, OTC cold preparations
myasthenia gravis, (stimulants) without physician
psychiatric disease, history approval. • Monitor B/P, pulse before
of severe anaphylaxis to taking medication. • Restrict salt, alcohol
allergens. intake.
Extended-Release:
Compensated HF

Trade Name Group Indication Side effect Precaution Interaction Adverse effects/ toxic Nursing care/ patient education
(Generic Name) reactions
(Carvedilol) Beta- Treatment of Frequent: Rash. Contraindications: DRUG: Antihypertensives, Hypotension (“first- BASELINE ASSESSMENT
Blockers hypertension, HF, History of diuretics may increase dose syncope”) may Obtain B/P immediately before each dose,
diabetic nephropathy, angioedema from hypotensive effects. May occur in pts with HF in addition to regular monitoring (be alert
post-MI for prevention Occasional: previous treatment increase lithium and in those who are to fluctuations). If hypotension occurs,
of ventricular failure. Pruritus, with ACE concentration, toxicity. severely place pt in supine position with legs
dysgeusia inhibitors, NSAIDs may decrease sodium/volume elevated. In pts with prior renal disease or
(altered taste). concomitant use antihypertensive effect. depleted. receiving dosages greater than 150 mg/
OFF-LABEL: Delays with aliskiren in pts Potassium-sparing Angioedema day, test urine for protein by dipstick
progression of with diabetes diuretics, potassium (swelling of method with first urine of day before
nephropathy and Rare: mellitus. supplements may cause face/tongue/lips), therapy begins and periodically thereafter.
reduces risk of Headache, hyperkalemia. hyperkalemia occur In pts with renal impairment, autoimmune
cardiovascular events in cough, rarely. disease, or taking drugs that affect
hypertensive pts with insomnia, Cautions: Renal Agranulocytosis, leukocytes or immune response, obtain
type 1 and type 2 dizziness, impairment. HERBAL: Ephedra, neutropenia noted in CBC before beginning therapy, q2wks for
diabetes. Treatment of fatigue, Hypertrophic ginseng, yohimbe may those with collagen 3 mos, then periodically thereafter
hypertensive crisis, paresthesia, cardiomyopathy worsen hypertension. vascular disease
rheumatoid arthritis; malaise, nausea, with outflow Garlic may increase (scleroderma,
hypertension secondary diarrhea or obstruction, before, antihypertensive effect. systemic lupus INTERVENTION/EVALUATION
to scleroderma renal constipation, during, or erythematosus), renal Assess skin for rash, pruritus. Assist with
crisis; diagnosis of dry mouth, immediately after impairment. ambulation if dizziness occurs. Monitor
aldosteronism; diagnosis tachycardia. major surgery. FOOD: Licorice may Nephrotic syndrome urinalysis for proteinuria. Monitor serum
of renal artery stenosis; Unstented cause sodium and water noted in those with potassium levels in those on concurrent
idiopathic edema; unilateral/ bilateral retention, hypokalemia. history of renal diuretic therapy. Monitor B/P, serum
Bartter’s syndrome; renal artery disease. BUN, creatinine, CBC. Discontinue
increases circulation in stenosis. medication, contact physician if
Raynaud’s syndrome. angioedema occurs

PATIENT/FAMILY TEACHING • Full


therapeutic effect of B/P reduction may
take several wks. • Skipping doses or
voluntarily discontinuing drug may
produce severe rebound hypertension. •
Limit alcohol intake. • Immediately
report if swelling of face, lips, or tongue;
difficulty breathing, vomiting, diarrhea,
excessive perspiration, dehydration,
persistent cough, sore throat, fever
occur. • Inform physician if pregnant or
planning to become pregnant. • Rise
slowly from sitting/lying position.

Trade Name Group Indication Side effect Precaution Interaction Adverse effects/ toxic Nursing care/ patient education
(Generic Name) reactions
Apresoline Anti- Management Occasional: Headache, Contraindicatio DRUG: Diuretics, lupus erythematosus– BASELINE ASSESSMENT Obtain BP, pulse
(hydralazine ) hypertensives of moderate anorexia, nausea, ns: Coronary other antihypertensives like reaction (fever, immediately before each dose, in addition to regular
to severe vomiting, diarrhea, artery disease, may increase facial rash, muscle/ monitoring (be alert to fluctuations).
hypertension. palpitations, mitral valvular hypotensive effect. joint aches,
tachycardia, angina rheumatic heart glomerulonephritis,
pectoris. disease, splenomegaly) . INTERVENTION/EVALUATION Monitor B/P,
OFF- dissecting aortic HERBAL: Ephedra, Severe orthostatic pulse, ANA titer. Monitor for headache, palpitations,
LABEL: aneurysm. ginseng, yohimbe may hypotension, skin tachycardia. Assess for peripheral edema of hands,
Hypertension Rare: Constipation, worsen hypertension. flushing, severe feet. Monitor daily pattern of bowel activity, stool
secondary to ileus, edema, Garlic may increase headache, myocardial consistency.
eclampsia, peripheral neuritis Cautions: Renal antihypertensive effect. ischemia, cardiac
preeclampsia. (paresthesia), dizziness, impairment, arrhythmias may
Treatment of muscle cramps, cerebrovascular develop. Profound PATIENT/FAMILY TEACHING
HF with anxiety, disease, positive FOOD: Any foods may shock may occur with • To reduce hypotensive effect, go from lying to
reduced hypersensitivity ANA titer, increase absorption. severe over dosage. standing slowly. • Report muscle/joint aches, fever
ejection reactions (rash, pulmonary (lupus-like reaction), flu-like symptoms. • Limit
fraction, urticaria, pruritus, hypertension alcohol use.
postoperative fever, chills, LAB VALUES: May
hypertension. arthralgia), nasal produce positive direct
congestion, flushing, Combest.
conjunctivitis.

Trade Name Group Indication Side effect Precaution Interaction Adverse effects/ Nursing care/ patient education
(Generic Name) toxic reactions
Zestril ACE Treatment of Frequent: Contraindications: DRUG: Diuretics may Excessive BASELINE ASSESSMENT Obtain
(Lisinopril) Inhibitors/Direct hypertension. Used Headache, History of angioedema increase effects. May hypotension B/P, apical pulse immediately before
Renin Inhibitors alone or in dizziness, postural from treatment with increase concentration, (“first-dose each dose, in addition to regular
combination with hypotension. ACE inhibitors, risk of toxicity of syncope”) may monitoring (be alert to fluctuations).
other idiopathic or hereditary lithium. NSAIDs may occur in pts with In pts with renal impairment,
antihypertensives. angioedema. decrease effects. HF, severe autoimmune disease, taking drugs
Adjunctive therapy Occasional: Chest Concomitant use with Potassium-sparing salt/volume that affect leukocytes or immune
in management of discomfort, fatigue, aliskiren in pts with diuretics, potassium depletion. response, CBC and differential count
heart failure. rash, abdominal diabetes. supplements may cause Angioedema should be performed before
Treatment of acute pain, nausea, hyperkalemia. May (swelling of face beginning therapy and q2wks for 3
MI within 24 hrs in diarrhea, upper increase hypoglycemic and lips), mos, then periodically thereafter.
hemodynamically respiratory Cautions: Renal effect of oral hyperkalemia
stable pts to infection. impairment, unstented hypoglycemic agents. occurs rarely.
improve survival. unilateral/bilateral renal Agranulocytosis, INTERVENTION/EVALUATION
Treatment of left artery stenosis, volume neutropenia may Assess for edema. Auscultate lungs
ventricular Rare: Palpitations, depletion, ischemic heart LAB VALUES: May be noted in pts for rales. Monitor I&O; weigh daily.
dysfunction tachycardia, disease, cerebrovascular increase serum BUN, with collagen Monitor daily pattern of bowel
following MI. peripheral edema, disease, severe aortic alkaline phosphatase, vascular disease activity, stool consistency. Assist
insomnia, stenosis, hypertrophic bilirubin, creatinine, (scleroderma, with ambulation if dizziness occurs.
paresthesia, cardiomyopathy. potassium, ALT, AST. systemic lupus Monitor B/P, renal function tests,
confusion, Concomitant use of May decrease serum erythematosus). WBC, serum potassium. If excessive
constipation, dry potassium supplements. sodium. May cause Nephrotic reduction in B/P occurs, place pt in
mouth, muscle positive ANA titer. syndrome may be supine position, feet slightly
cramps. noted in pts with elevated.
history of renal
disease PATIENT/ FAMILY TEACHING
• To reduce hypotensive effect, go
from lying to standing slowly. •
Limit alcohol intake. • Report
vomiting, diarrhea, diaphoresis,
swelling of face/lips/tongue,
difficulty in breathing, persistent
cough. • Limit salt intake
Trade Name Group Indication Side effect Precaution Interaction Adverse effects/ Nursing care/ patient education
(Generic Name) toxic reactions
Herbesser Calcium Treatment of angina due Frequent: Contraindications: PO: DRUG: Beta blockers, Abrupt BASELINE ASSESSMENT Record
(diltiazem) Antagonists,Anti to coronary artery spasm Peripheral edema, Acute MI, pulmonary digoxin may have additive withdrawal may onset, type (sharp, dull, squeezing),
-Anginal Drugs (Prinzmetal’s variant dizziness, light- congestion, effect on prolonging AV increase radiation, location, intensity,
angina), chronic stable headedness, hypersensitivity to conduction. May increase frequency, duration of anginal pain,
angina (effortassociated headache, diltiazem or other concentration, risk of duration of precipitating factors (exertion,
angina). bradycardia, calcium channel blockers, toxicity with angina, HF; emotional stress). Assess baseline
asthenia. 2 - or 3 -degree AV
nd rd
carbamazepine, secondand third- renal/hepatic function tests. Assess
block (except in presence benzodiazepines. May degree AV block B/P, apical pulse immediately before
Extended-release: of pacemaker), severe increase serum digoxin occur rarely. drug is administered.
Treatment of essential Occasional: hypotension (< 90 concentration. Rifampin Overdose
hypertension, angina. Nausea, mmHg, systolic), sick may decrease produces nausea,
constipation, sinus syndrome. concentration/effects. May drowsiness,
flushing, EKG increase concentration of confusion, INTERVENTION/EVALUATION
changes. statins and risk of slurred speech, Assist with ambulation if dizziness
Cardizem LA: IV: Sick sinus syndrome myopathy/rhabdomyolysis. profound occurs. Assess for peripheral edema.
Treatment of chronic or 2 - or 3 degree block
nd rd
bradycardia. Monitor pulse rate for bradycardia.
stable angina. Rare: Rash, (except with functioning LAB VALUES: EKG: Antidote: Assess B/P, renal function, LFT,
micturition pacemaker), cardiogenic May increase PR interval. Glucagon, EKG with IV therapy. Question for
disorder shock, administration of insulin drip with asthenia headache.
Parenteral: Temporary (polyuria, IV beta blocker within continuous
control of rapid nocturia, dysuria, several hours, atrial calcium infusion PATIENT/FAMILY TEACHING
ventricular rate in atrial frequency of fibrillation/flutter • Do not abruptly discontinue
fibrillation/flutter. Rapid urination), associated with accessory medication. • Compliance with
conversion of abdominal bypass tract. therapy regimen is essential to
paroxysmal discomfort, control anginal pain. • To avoid
supraventricular drowsiness. hypotensive effect, go from lying to
tachycardia (PSVT) to Cautions: Renal/hepatic standing slowly. • Avoid tasks
normal sinus rhythm. impairment, HF, that require alertness, motor skills
concurrent use with beta- until response to drug is
blocker, hypertrophic established. • Report palpitations,
OFFLABEL: Stable obstructive shortness of breath, pronounced
narrow complex cardiomyopathy dizziness, nausea, constipation. •
tachycardia, recurrent Avoid alcohol (may increase risk of
SVT, pediatric hypotension or vasodilation).
hypertension,
hypertrophic
cardiomyopathy.

Trade Name Group Indication Side effect Precaution Interaction Adverse effects/ Nursing care/ patient education
(Generic toxic reactions
Name)
Norvasc Calcium hypertension, Frequent: Peripheral Contraindications: DRUG: May increase level of excessive BASELINE ASSESSMENT
(Amlodipine) Antagonists, chronic stable edema, headache, None known. simvastatin. Azole antifungals, peripheral Assess baseline renal/hepatic function
Anti-Anginal angina, flushing. cyclosporine protease inhibitors may vasodilation, tests, B/P, apical pulse.
Drugs vasospastic increase marked
(Prinzmetal’s or Cautions: Hepatic concentration. Carbamazepine, hypotension INTERVENTION/EVALUATION
variant) angina. Occasional: impairment, aortic rifampin may decrease level/effect. with reflex Assess B/P (if systolic B/P is less than
May be used Dizziness, stenosis, 90 mmHg, withhold medication.Assess
alone or with palpitations, nausea, hypertophic tachycardia, for peripheral edema behind medial
other unusual fatigue or cardiomyopathy HERBAL: St. John’s wort may syncopy. malleolus (sacral area in bedridden
antihypertensives weakness (asthenia). with outflow tract decrease concentration. Ephedra, pts). Assess skin for flushing.
or antianginals. obstruction. yohimbe may worsen hypertension. Question for headache, asthenia.
Rare: Chest pain, Garlic may increase PATIENT/FAMILY TEACHING
bradycardia, antihypertensive effect. • Do not abruptly discontinue
orthostatic medication. • Compliance with
hypotension. FOOD: Grapefruit products may therapy regimen is essential to control
increase concentration, hypotensive hypertension.• Avoid tasks that
effects. LAB VALUES: May require alertness,motor skills until
increase hepatic enzyme levels. response to drug is established. • Do
not ingest grapefruit products.

Trade Name Group Indication Side effect Precaution Interaction Adverse effects/ Nursing care/ patient education
(Generic Name) toxic reactions
Warfarin Anticoagulants, Prophylaxis, Occasional: GI Contraindications: DRUG: Bleeding BASELINE ASSESSMENT
Antiplatelets & treatment of distress (nausea, Hemorrhagic tendencies (e.g., Amiodarone, azole complications Cross-check dose with coworker.
Fibrinolytics thromboembolic anorexia, cerebral aneurysms), surgery antifungals, ranging from local Determine INR before administration
(Thrombolytics) disorders and embolic abdominal of eye or CNS, neurosurgical cimetidine, ecchymoses to and daily following therapy initiation.
complications arising cramps, procedures, open wounds, disulfram, major hemorrhage When stabilized, follow with INR
from atrial fbrillation diarrhea). severe hypertension, spinal fluvoxamine, may occur. determination q4–6wks. Obtain
or valve replacement. puncture procedures, sulfamethoxazole- Antidote: Vitamin
Risk reduction of uncontrolled bleeding, ulcers, trimethoprim, K. Amount based genotyping prior to initiating therapy if
systemic embolism unreliable or noncompliant pt, levothyroxine, on INR, signifcance available.
following MI (e.g., Rare: unsupervised senile or metronidazole, of bleeding. Range:
recurrent MI, stroke). Hypersensitivity psychotic pt, blood dyscrasias, NSAIDs, 2.5–10 mg given
reaction pericarditis or pericardial omeprazole, platelet orally or slow IV INTERVENTION/EVALUATION
(dermatitis, effusion, pregnancy (except in aggregation infusion. Monitor INR reports diligently. Assess
OFF-LABEL: urticaria), esp. women with inhibitors, Hepatotoxicity, Hct, platelet count, ALT, AST,
Adjunct treatment in in those mechanical heart valves at salicylates, blood dyscrasias, urine/stool for occult blood. Be alert to
transient ischemic sensitive to high risk for thrombolytic necrosis, vasculitis, complaints of abdominal/back pain,
attacks. aspirin thromboembolism), bacterial agents, thyroid local thrombosis severe headache (may be sign of
endocarditis, threatened hormones may occur rarely. hemorrhage). Decrease in B/P, increase
abortion. increase effect. in pulse rate may be sign of hemorrhage.
Griseofulvin, Question for increase in amount of
hepatic enzyme menstrual discharge. Assess peripheral
Cautions: Active tuberculosis, inducers (e.g., pulses; skin for ecchymoses,
acute infection, diabetes, rifampin), vitamin petechiae. Check for excessive bleeding
heparin-induced K may from minor cuts, scratches. Assess gums
thrombocytopenia, pts at risk decrease effects. for erythema, gingival bleeding.
for hemorrhage, moderate to Alcohol may
severe renal impairment, enhance
moderate to severe anticoagulant PATIENT/ FAMILY TEACHING
hypertension, thyroid disease, effect. • Take medication at same time each
polycythemia vera, vasculitis, day. • Blood levels will be monitored
open wound, menstruating and routinely. • Do not take, discontinue any
postpartum women, other medication except on advice of
indwelling catheters. physician. • Avoid alcohol, aspirin,
drastic dietary changes. • Do not change
from one brand to another. • Consult
with physician before surgery, dental
work. • Urine may become red-orange. •
Avoid, minimize signifcant bodily
trauma.

Trade Name Group Indication Side effect Precaution Interaction Adverse effects/ Nursing care/ patient education
(Generic Name) toxic reactions
Aspirin Nonsteroidal Treatment of mild to Occasional: GI contraindications: DRUG: Alcohol, mucosal lesions. BASELINE ASSESSMENT
(Acetylsalicylic Anti- moderate pain, fever. distress Hypersensitivity to NSAIDs may Dehydrated, Do not give to children or teenagers
Acid) Inflammatory Reduces inflammation (abdominal salicylates, NSAIDs. increase febrile children who
Drugs related to rheumatoid distention, Asthma, rhinitis, nasal risk of GI effects may experience have or recently had viral infections
(NSAIDs) / arthritis, juvenile arthritis, cramping, polyps; inherited or (e.g., ulceration). aspirin toxicity (increases risk of Reye’s syndrome).
Anticoagulants, osteoarthritis, rheumatic heartburn, mild acquired bleeding Antacids, urinary quickly. Reye’s Do not use if vinegar-like odor is
Antiplatelets & fever. Used as platelet nausea); allergic disorders; use in alkalinizers syndrome, noted (indicates chemical
Fibrinolytics aggregation inhibitor in the reaction children (<16 yrs) for increase characterized by breakdown). Assess type, location,
(Thrombolytics) prevention of transient (bronchospasm, viral infections. excretion. persistent duration of pain, inflammation.
ischemic attacks (TIAs), pruritus, urticaria). Anticoagulants, vomiting, signs of Inspect appearance of affected joints
cerebral thromboembolism, heparin, brain dysfunction, for
MI or reinfarction. Cautions: thrombolytics, may occur in immobility, deformities, skin
Platelet/bleeding rivaroxaban, children taking condition.
disorders, severe renal/ ticagrelor increase aspirin with Therapeutic serum level for
OFF-LABEL: Prevention of hepatic impairment, risk of bleeding. recent viral antiarthritic effect: 20–30 mg/dL
pre-eclampsia; alternative dehydration, erosive infection (toxicity
therapy for preventing gastritis, peptic ulcer HERBAL: (chickenpox, occurs if level is >30 mg/dL)
thromboembolism disease, sensitivity Avoid 當歸, common cold, or
associated with atrial to tartrazine dyes, feverfew, garlic, flu). Low-grade
fibrillation when warfarin elderly (chronic use of ginger, ginkgo, aspirin toxicity PATIENT/FAMILY TEACHING
cannot be used; pericarditis doses 325 mg or ginseng, green characterized by • Do not, chew, crush, dissolve, or
associated with MI; greater). Avoid use in tea, horse tinnitus, divide enteric-coated tablets.
prosthetic valve pregnancy, especially chestnut, red generalized • Avoid alcohol. • Report tinnitus or
thromboprophylaxis. third trimester. clover (possess pruritus (may be persistent abdominal, GI pain,
Adjunctive treatment of antiplatelet severe), headache, bleeding. • Therapeutic anti
Kawasaki’s disease. activity). dizziness, inflammatory effect noted in 1–3
Complications associated flushing, wks • Behavioral changes, persistent
with autoimmune disorders; LAB VALUES: tachycardia, vomiting may be early signs of
colorectal cancer May alter serum hyperventilation, Reye’s syndrome
ALT, AST, diaphoresis, thirst.
alkaline Marked
phosphatase, uric toxicity
acid; prolongs characterized by
prothrombin time hyperthermia,
(PT), bleeding restlessness,
time. May seizures,
decrease serum abnormal
cholesterol, breathing
potassium, T3, patterns,
T4. respiratory
failure, coma

Trade Name Group Indication Side effect Precaution Interaction Adverse effects/ Nursing care/ patient education
(Generic toxic reactions
Name)
Plavix Anticoagulants, Unstable Frequent : Skin Contraindications: DRUG: Aspirin, NSAIDs, Agranulocytosis, BASELINE ASSESSMENT
(Clopidogrel) Antiplatelets & angina/non–ST- disorders. Active bleeding warfarin may increase risk of aplastic anemia/ Obtain baseline chemistries, platelet
Fibrinolytics segment elevation (e.g. peptic ulcer, bleeding. Proton pump inhibitors pancytopenia, count, PFA level. Perform platelet
(Thrombolytics) MI. ST-segment intracranial (e.g., omeprazole) may decrease thrombotic counts
elevation, acute MI. hemorrhage). effcacy, increase risk of thrombocytopenic before drug therapy, q2days during first
Recent MI, stroke, or Occasional: Upper cardiovascular events. purpura (TTP) wk of treatment, and weekly thereafter
established resp tract infection, Cautions: Severe occur rarely. until therapeutic maintenance dose is
peripheral arterial chest pain, flu-like hepatic/renal Hepatitis, reached. Abrupt discontinuation of drug
disease. symptoms, impairment, pts at HERBAL: Cat’s claw, dong quai, hypersensitivity therapy produces elevated platelet count
headache, risk of increased evening primrose, feverfew, reaction, within 5 days.
dizziness, bleeding garlic, ginger, ginkgo, ginseng, anaphylactoid
OFF-LABEL: Graft arthralgia. (e.g., trauma), green tea, red clover may have reaction have INTERVENTION/EVALUATION
patency (saphenous concurrent use of additive antiplatelet effects. been reported Monitor platelet count for evidence of
vein), stable anticoagulants. thrombocytopenia. Assess Hgb, Hct,
coronary artery Rare: Fatigue, Avoid concurrent WBC; serum ALT, AST, bilirubin,
disease (in edema, use of CYP2C19 FOOD: Grapefruit products may BUN,
combination with hypertension, inhibitors, decrease effects. creatinine; signs/symptoms of hepatic
aspirin). Initial abdominal pain, omeprazole. insufficiency during therapy.
treatment of acute dyspepsia,
coronary syndrome diarrhea, LAB VALUES: May increase PATIENT/FAMILY TEACHING
in pts allergic to nausea, epistaxis, serum bilirubin, ALT, AST, • It may take longer to stop bleeding
aspirin dyspnea, rhinitis. cholesterol, uric acid. May during drug therapy. • Report any
decrease unusual bleeding. • Inform physicians,
neutrophil count, platelet count. dentists if clopidogrel is being taken,
esp.
before surgery is scheduled or before
taking any new drug

Trade Name Group Indication Side effect Precaution Interaction Adverse effects/ Nursing care/ patient education
(Generic toxic reactions
Name)
Effient Anticoagulants, of thrombotic Occasional (4-8%): Contraindications: Active DRUG: Aspirin, Major bleeding BASELINE ASSESSMENT
(Prasugrel) Antiplatelets & cardiovascular HT, minor bleeding, bleeding, prior transient NSAIDs, warfarin (intracranial Obtain baseline vital signs, CBC,
Fibrinolytics events (MI, CVA, stent headache, back pain, may hemorrhage, EKG, hepatic function tests.
(Thrombolytics) thrombosis) in pts with dyspnea, nausea, ischemic attack (TIA), increase risk of epistaxis, GI
acute coronary syndrome dizziness. CVA. bleeding. bleeding,
(unstable angina, non– hemoptysis, INTERVENTION/EVALUATION
ST-segment elevation Cautions: Pts who undergo subcutaneous Monitor vital signs for changes in
MI, ST-segment MI) who Rare (<4%): Cough, coronary artery bypass HERBAL: Cat’s hematoma, B/P, pulse. Assess for signs of
are to be managed hypotension, fatigue, graft (CABG) after claw, dong quai, postprocedural unusual bleeding or hemorrhage,
with percutaneous noncardiac chest pain, receiving prasugrel, pts at evening primrose, hemorrhage, pain. Monitor platelet count, LFT,
coronary intervention bradycardia, rash, risk for bleeding (age 75 feverfew, garlic, retroperitoneal EKG for changes from baseline.
(PCI). pyrexia, peripheral yrs or older, body weight < ginger, ginseng, hemorrhage,
edema, extremity 60 kg, recent green tea, horse retinal PATIENT/FAMILY TEACHING
pain, diarrhea. trauma/surgery, recent GI chestnut, red clover hemorrhage) has • It may take longer to stop minor
OFF-LABEL: Initial bleeding or active peptic may have additive been reported. bleeding during drug therapy.
treatment of unstable ulcer disease, severe platelet effects. Report unusual bleeding/bruising,
angina, STEMI in pts hepatic impairment). Ginkgo blood noted in stool or urine,
undergoing PCI with biloba may increase chest/back pain, extremity pain.
allergy or major GI risk of bleeding. Severe • Monitor for dyspnea.
intolerance to aspirin thrombocytopenia, •Report fever, weakness, extreme
anemia, abnormal skin paleness, purple skin patches,
LAB VALUES: May hepatic function, yellowing of
decrease Hgb, Hct, anaphylactic skin or eyes, changes in mental
WBC, platelet count. reaction, status. •Do not discontinue drug
May increase bleeding angioedema, atrial therapy without physician
time, serum approval. •Inform physicians,
cholesterol, ALT, fbrillation occur dentists before undergoing any
AST. rarely. invasive procedure or surgery.

Overdosage may
require platelet
transfusion to
restore clotting
ability

Trade Name Group Indication Side effect Precaution Interaction Adverse effects/ toxic Nursing care/ patient education
(Generic Name) reactions
Brilinta Anticoagulants, Reduction of Occasional Contraindications: antiplatelets, Life-threatening events BASELINE ASSESSMENT
(ticagrelor) Antiplatelets & thrombolytic (7-13%): History of NSAIDs may including intracranial Obtain CBC, serum chemistries, renal
Fibrinolytics cardiovascular Dyspnea, intracranial increase risk of bleeding, epistaxis, function, LFT, digoxin level if applicable.
(Thrombolytics) events in conjunction headache. hemorrhage, active bleeding. May intrapericardial bleeding with Question for history of bleeding, stomach
with aspirin in pts pathologic bleeding, increase cardiac tamponade, ulcers, colon polyps, head trauma, cardiac
with acute coronary severe hepatic concentration of hypovolemic shock requiring arrhythmias, unstable angina, recent MI,
syndrome (ACS) Rare (3- impairment. digoxin, simvastatin, vasopressive support, or blood hepatic impairment, hypertension, stroke.
including unstable 5%): lovastatin. transfusion reported. Pts with Receive full medication history including
angina (UA), non-ST Cough, history of sick sinus herbal products. Question for history of
elevation myocardial dizziness, Cautions: FOOD: Grapefruit syndrome, second- or third- COPD, chronic bronchitis, emphysema,
infarction (STEMI), nausea, Moderate hepatic products may degree AV block, bradycardic asthma, exertional dyspnea.
or STEMI. diarrhea, impairment, renal increase potential for syncope have increased risk of
back pain, impairment, history bleeding. bradycardia. May induce
fatigue. of hyperuricemia or episodes of atrial fbrillation, INTERVENTION/EVALUATION
OFF-LABEL: Initial gouty arthritis. Pts at hypotension, hypertension. Routinely screen for bleeding. Assess skin
treatment of unstable increased risk of LAB VALUES: May Gynecomastia reported in less for bruising, hematoma. Monitor renal
angina, non-STEMI in bradycardia, increase serum uric than 1% of men. function, uric acid, digoxin levels if
pts with allergy to concurrent use of acid, creatinine applicable. Report hematuria, epistaxis,
aspirin or major GI strong coffee-ground emesis, black/tarry stools.
intolerance to aspirin. CYP3A4 inhibitors Monitor EKG for chest pain, shortness of
or inducers, elderly. breath, syncope
(Recommend holding
dose 5 days before
planned surgery if PATIENT/FAMILY TEACHING
applicable.) • It may take longer to stop bleeding
during therapy. • Do not vigorously blow
nose. • Use soft toothbrush, electric razor
to decrease risk of bleeding. • Immediately
report bloody stool, urine, or nosebleeds. •
Report all newly prescribed medications. •
Inform physician of any planned dental
procedures or surgeries.

Respiratory system
Trade Name Group Indication Side effect Precaution Interaction Adverse effects/ toxic Nursing care/ patient education
(Generic Name) reactions
Ventolin Antiasthmatic Treatment or Frequent: Headache; Contraindications: DRUG: Beta-adrenergic Excessive BASELINE ASSESSMENT
(Albuterol) & COPD prevention of restlessness, History of blocking agents (beta- sympathomimetic Assess lung sounds, pulse, B/P, color,
(salbutamol) Preparations bronchospasm nervousness, tremors; hypersensitivity to blockers) antagonize stimulation may character of sputum noted. Offer
due to nausea; dizziness; throat sympathomimetics. effects. May produce produce palpitations, emotional support (high incidence of
reversible dryness and irritation, bronchospasm. exectopy, tachycardia, anxiety due to difficulty in breathing and
obstructive pharyngitis; B/P Atomoxetine, MAOIs, chest pain, slight sympathomimetic response to drug).
airway changes, including Cautions: tricyclic antidepressants increase in B/P
disease, hypertension; Hypertension, may potentiate followed by substantial
prevention of heartburn, transient cardiovascular cardiovascular effects. decrease, chills, INTERVENTION/EVALUATION
exercise wheezing. disease, May increase effects of diaphoresis, blanching Monitor rate, depth, rhythm, type of
induced hyperthyroidism, loop diuretics (produce of skin. Too-frequent respiration; quality and rate of pulse;
bronchospasm. diabetes mellitus, hypokalemia), or excessive use may EKG; serum potassium, glucose; ABG
Occasional): Insomnia, HF, convulsive sympathomimetics lead to decreased determinations. Assess lung sounds for
asthenia, altered disorders, (increase CNS bronchodilation wheezing
taste. Inhalation: Dry, glaucoma, stimulation). effectiveness and (bronchoconstriction), rales.
irritated mouth or hypokalemia, FOOD: Limit caffeine severe, paradoxical
throat; cough; bronchial arrhythmias. (may cause CNS bronchoconstriction. PATIENT/FAMILY TEACHING
irritation. stimulation). •Follow guidelines for proper use of
inhaler. •Increase fluid intake (decreases
lung secretion viscosity). •Do not take
Rare: LAB VALUES: May more than 2 inhalations at any one time
Drowsiness, diarrhea, increase blood glucose (excessive use may produce paradoxical
dry mouth, flushing, level. May decrease bronchoconstriction or decreased
diaphoresis, anorexia. serum potassium level. bronchodilating effect).
•Rinsing mouth with water immediately
after inhalation may
prevent mouth/throat dryness.
•Avoid excessive use of caffeine
derivatives (chocolate, coffee, tea, cola,
cocoa)
Trade Name Group Indication Side effect Precaution Interaction Adverse Nursing care/ patient education
(Generic Name) effects/ toxic
reactions
Atrovent Antiasthmatic & Inhalation, Frequent: Contraindications: DRUG: Worsening of BASELINE ASSESSMENT
(ipratropium) COPD Nebulization: Inhalation : Cough, dry mouth, History of Anticholinergics, angle-closure Offer emotional support (high
Preparations Maintenance treatment headache, nausea. hypersensitivity to medications glaucoma, incidence of anxiety due to difficulty
of bronchospasm due to Nasal: Dry nose/mouth, headache, atropine. with anticholinergic acute eye pain, in breathing,
COPD, bronchitis, nasal irritation. properties may hypotension sympathomimetic response to drug).
(anticholinergics) emphysema, asthma. increase toxicity. occur rarely
Not indicated for Occasional: Inhalation (2%): Cautions: INTERVENTION/EVALUATION
immediate Dizziness, transient increased Narrow-angle Monitor rate, depth, rhythm, type of
bronchospasm relief. bronchospasm. glaucoma, respiration; quality, rate of pulse.
prostatic Assess lung sounds for rhonchi,
Rare (less than 1%): Inhalation: hypertrophy, wheezing, rales. Monitor ABGs.
Nasal Spray: Hypotension, bladder Observe lips, fingernails
Symptomatic insomnia,metallic/unpleasant neck obstruction, for cyanosis (blue or dusky color in
relief of rhinorrhea taste, palpitations, urinary myasthenia light skinned pts; gray in dark-
associated with the retention. gravis. skinned pts). Observe for retractions
common cold and (clavicular, sternal, intercostal),
allergic/nonallergic Nasal: Diarrhea, constipation, dry hand tremor. Evaluate for clinical
rhinitis throat, abdominal pain, nasal improvement (quieter, slower
congestion. respirations, relaxed facial
** expression, cessation of retractions).
Monitor for
improvement of rhinorrhea.

PATIENT/ FAMILY TEACHING


• Increase fluid intake (decreases
lung secretion viscosity). • Do not
take > 2 inhalations at any one time
(excessive use may produce
paradoxical bronchoconstriction,
decreased bronchodilating effect). •
Rinsing mouth with water
immediately after inhalation may
prevent mouth and throat dryness.
• Avoid excessive use of caffeine
derivatives (chocolate, coffee, tea,
cola, cocoa).
Trade Name Group Indication Side effect Precaution Interaction Adverse effects/ Nursing care/ patient
(Generic Name) toxic reactions education
Becloforte Antiasthmatic & used in the treatment irritation or drying of the CONTRAINDICATIONS: Unknown Unknown PATIENT/ FAMILY
(beclometasone) COPD of obstructive airway mouth and throat, coughing, Hypersensitivity to any of TEACHING
Preparations diseases or difficulty speaking the ingredients of this • Increase fluid intake
preparation. Do not rush stages 2 and 3. It
immediate medical attention Acute status asthmaticus. is important to breathe in
include rashes, difficulty slowly through your mouth
breathing, swelling in the just before pressing the
face and lower extremities, canister. -To be sure of using
extended cold or other your inhaler properly, you
similar infections and may initially practice these
muscle weakness steps in front of a mirror.
-Clean your inhaler at least
once a week.
Trade Name Group Indication Side effect Precaution Interaction Adverse effects/ Nursing care/ patient education
(Generic Name) toxic reactions
Piriton Antihistamines Adult : PO Allergic CNS depression, Contraindication: Drug: Potentiates Somnolence, PATIENT EDUCATION
(chlorphenamine) & Antiallergics conditions 4 mg 4-6 sedation, drowsiness, Hypersensitivity, neonates sedative effect of anticholinergic -No work that requires attention.
hrly. Max: 24 lassitude, dizziness. GI psychotropic drugs e.g. effects (ie, -Avoid alcohol
mg/day. upsets, anorexia, or Caution: barbiturates, hypnotics, mydriasis,
IV/IM/SC Adjunct increased appetite, Elderly, pylori duodenal opioid analgesics, flushing, fever,
in anaphylactic epigastric pain, obstruction, angle-closure anxiolytics and dry mouth, and
shock10-20 mg. blurring of vision, glaucoma, urinary antipsychotics. tachycardia,
Total max: 40 dysuria, dryness of retention, prostatic Interaction with alcohol
mg/day. mouth, tightness in hyperplasia, epilepsy, renal could be dangerous
chest, hypotension, and hepatic impairment. (sedation/excitation).
muscular weakness, May affect performance of
tinnitus, euphoria, skilled tasks. BPH, bladder
headache, paradoxical neck obstruction,
CNS stimulation. hypertension. Pregnancy,
Potentially Fatal: CV lactation.
collapse and
respiratory failure.
Trade Name Group Indication Side Precaution Interaction Adverse effects/ toxic reactions Nursing care/ patient education
(Generic Name) effect
MES (Mixture Analges treat coughs and congestion caused by Nausea Contraindication: Not known Not known BASELINE ASSESSMENT
Expectorant ics the common cold, bronchitis, and other or vomi allergic to it Assess history of allergy
Stimulant) (Opioid breathing illnesses. This product is usually ting ma
) not used for y occur. PATIENT EDUCATION
ongoing cough from smoking or long- -Encourage sufficient fluid intake
term breathing problems (such as chronic
bronchitis, emphysema) unless directed by
your doctor. Guaifenesin is an expectorant.
It works by thinning and
loosening mucus in the airways, clearing
congestion, and making breathing easier.
Trade Name Group Indication Side effect Precaution Interaction Adverse Nursing care/ patient education
(Generic effects/ toxic
Name) reactions
Phensedyl Antitussive antihistamine, narcotic analgesic pts not in severe pain Contraindications: Respiratory DRUG: Alcohol, Chronic use BASELINE ASSESSMENT
(Codeine) and sympathomimetic for treatment may experience depression in absence of other CNS may result in Analgesic:
Expectorants. of cough. Relief of mild to dizziness, nausea, resuscitative equipment, acute depressants may paralytic Assess onset, type, location,
moderate pain. vomiting, hypotension or severe bronchial asthma or increase CNS, ileus. duration of pain. Effect of
more hypercarbia, paralytic ileus. respiratory Overdose may medication is reduced if full pain
OFFLABEL: frequently than those in Postoperative pain depression, produce response recurs before
Short-term relief of cough. supine position or with management in children hypotension. cold/clammy next dose. Antitussive: Assess
severe pain. following tonsillectomy Anticholinergics skin,
/adenoidectomy. may increase risk confusion, type, severity, frequency of cough,
of urinary seizures, sputum production.
Frequent: Constipation, retention, severe decreased
drowsiness, nausea, Cautions: Adrenal constipation. B/P,
vomiting. insuffciency, biliary MAOIs may restlessness, INTERVENTION/EVALUATION
tract impairment, CNS produce a severe, pinpoint Monitor daily pattern of bowel
depression/coma, sometimes fatal pupils, activity, stool consistency. Increase
Occasional: Paradoxical morbid obesity, prostatic reaction (reduce bradycardia, fluid intake, environmental
excitement, confusion, hyperplasia, urinary stricture, dosage to ¼ usual respiratory humidity to improve viscosity of
palpitations, facial thyroid dysfunction, severe dose). depression, lung secretions. Initiate deep
flushing renal/hepatic impairment, decreased breathing, coughing exercises.
COPD, respiratory disease, LOC, Assess for clinical improvement;
cardiovascular disease, HERBAL: severe record onset of relief of pain,
** hypovolemia, GI obstruction, St. John’s wort weakness. cough.
head injury, elevated may decrease Tolerance to PATIENT/FAMILY TEACHING
intracranial pressure, history of concentration. drug’s • Change positions slowly to avoid
drug abuse, patients with 2 or Gotu kola, kava analgesic orthostatic hypotension. • Avoid
more copies of variant kava, SAMe, effect, tasks that require alertness, motor
CYP2D6*2 allele (may have St. John’s wort, physical skills until response to drug is
extensive conversion to valerian may dependence established.
morphine). increase CNS may occur • Tolerance,dependence may occur
depression. with chronic with prolonged use of high
use. dosages. • Avoid alcohol
LAB VALUES:
May increase
serum amylase,
lipase.

Central nervous system


Trade Name Group Indication Side effect Precaution Interaction Adverse effects/ toxic Nursing care/ patient education
(Generic Name) reactions
Panadol Analgesics Relief of mild to Rare: Contraindications: Severe DRUG: Alcohol (chronic Early Signs of BASELINE ASSESSMENT
(acetaminophen/ (Non-Opioid) moderate pain, Hypersensitivity hepatic impairment or severe use), hepatotoxic Acetaminophen If given for analgesia, assess onset,
Paracetamol) & Antipyretics fever. reaction. active liver disease (Ofrmev). medications (e.g., Toxicity: Anorexia, type, location, duration of pain.
phenytoin), nausea, diaphoresis, Effect of medication is reduced if
hepatic enzyme inducers fatigue within first 12– full pain response
IV: (Additional) (e.g., phenytoin, rifampin) 24 hrs. Later Signs of recurs prior to next dose. Assess
Management of Cautions: Sensitivity to may increase risk of Toxicity: Vomiting, for fever. Assess alcohol usage.
moderate to acetaminophen; severe renal hepatotoxicity with right upper quadrant
severe pain when impairment; alcohol prolonged high dose tenderness, elevated INTERVENTION/EVALUATION
combined with dependency, hepatic or single toxic dose. May hepatic function tests Assess for clinical improvement
opioid analgesia impairment, or active hepatic increase risk of within 48–72 hrs after and relief of pain, fever.
disease; chronic malnutrition bleeding with warfarin ingestion. Antidote: Therapeutic serum
and hypovolemia (Ofrmev); with chronic, Acetylcysteine level: 10–30 mcg/ml; toxic serum
G6PD defciency (hemolysis high-dose use. level: greater than 200 mcg/ml. Do
may occur). Limit not exceed maximum daily
dose to less than 4 g/day recommended dose: 4000mg/day.
LAB VALUES: May
increase serum ALT, AST, PATIENT/FAMILY TEACHING
** bilirubin, prothrombin • Consult physician for use in
levels (may indicate children <2 yrs, oral use longer
hepatotoxicity). than 5 days (children) or longer
than 10 days (adults), or fever
lasting longer than 3 days.
•Severe/recurrent pain or high
/continuous fever may indicate
serious illness. • Advise not to
take more than 4000mg /24-hr
period. “如每粒 500mg,即每天
最多吃 8 粒)每次不能超過
1000mg(即約兩粒)”
•Many nonprescription
combination products contain
acetaminophen. Avoid alcohol.

Trade Name Group Indication Side effect Precaution Interaction Adverse effects/ toxic Nursing care/ patient education
(Generic reactions
Name)
Tramadol Analgesics Management of Frequent: Dizziness, Contraindications: Ultram DRUG: Alcohol, other CNS Seizures reported in pts BASELINE ASSESSMENT
(Ultram) (Opioid) moderate to vertigo, nausea, ER: depressants may increase CNS receiving Assess onset, type, location,
moderately constipation, Acute alcohol intoxication, depression. Carbamazepine tramadol within duration of pain. Assess drug
severe pain. concurrent use of centrally decreases concentration/effects. recommended dosage history, esp. carbamazepine,
Extended- headache, acting analgesics, CYP2D6 inhibitors (e.g., range. May have analgesics, CNS depressants,
Release: drowsiness. hypnotics, opioids, paroxetine), CYP3A4 inhibitors prolonged duration of MAOIs. Review past medical
Around-the- psychotropic drugs, (e.g., erythromycin), triptans, action, cumulative history, esp. epilepsy, seizures.
clock Occasional: hypersensitivity to opioids. selective serotonin reuptake effect in pts with Assess renal function, LFT.
management of Vomiting, ConZip, Severe/ acute inhibitors (SSRIs), tricyclic hepatic/renal
moderate to pruritus, CNS bronchial asthma, antidepressants may increase impairment, serotonin INTERVENTION/EVALUATION
moderately stimulation (e.g., hypercapnia, signifcant risk of seizures, risk of syndrome (agitation, Monitor pulse, B/P, renal/hepatic
severe pain for nervousness, anxiety, respiratory depression. serotonin syndrome. hallucinations, function. Assist with ambulation if
extended period. agitation, tremor, tachycardia, dizziness, vertigo occurs. Dry
euphoria, mood hyperreflexia). crackers, cola may relieve nausea.
swings, Caution: CNS depression, LAB VALUES: May increase Palpate bladder for urinary
hallucinations), anoxia, advanced hepatic serum creatinine, ALT, AST. retention. Monitor daily pattern of
asthenia, diaphoresis, cirrhosis, respiratory May decrease Hgb. May cause bowel activity, stool consistency.
dyspepsia, dry depression, elevated ICP, proteinuria. Sips of water may relieve dry
mouth, diarrhea. history of seizures or risk mouth. Assess for clinical
Rare: Malaise, for seizures, hepatic/renal improvement, record onset of
vasodilation, impairment, treatment of relief of pain.
anorexia, flatulence, acute abdominal conditions,
rash, blurred vision, opioid-dependent pts, head PATIENT/FAMILY TEACHING
urinary retention/ injury, myxedema, • May cause dependence. • Avoid
frequency, hypothyroidism, alcohol, OTC medications
menopausal hypoadrenalism, pregnancy. (analgesics, sedatives). • May
symptoms. Avoid use in pts who are cause drowsiness, dizziness,
suicidal or addiction prone, blurred vision. • Avoid tasks
emotionally disturbed, requiring alertness, motor skills
depressed, heavy alcohol until response to drug is
users, elderly, debilitated established. • Report severe
pts. constipation, difficulty breathing,
excessive sedation, seizures,
muscle weakness, tremors, chest
** pain, palpitations.

Trade Name Group Indication Side effect Precaution Interaction Adverse effects/ Nursing care/ patient education
(Generic Name) toxic reactions
Valium Anxiolytics, Short-term relief of Frequent: Pain with IM Contraindications: Acute DRUG: Alcohol, IV route may BASELINE ASSESSMENT
(diazepam) Anticonvulsants, anxiety symptoms, injection, narrow-angle CNS depressants produce pain, Assess B/P, pulse, respirations
Hypnotics & relief of acute drowsiness, fatigue, ataxia. glaucoma, severe may increase CNS swelling, immediately before administration.
Sedatives alcohol withdrawal. respiratory depression, depression. CYP3A4 thrombophlebitis, Anxiety: Assess autonomic
Adjunct for relief of severe hepatic inducers (e.g., carpal tunnel response (cold, clammy hands,
acute Occasional: Slurred speech, insufficiency, sleep apnea carbamazepine, syndrome. Abrupt diaphoresis), motor response
musculoskeletal orthostatic hypotension, syndrome, myasthenia rifampin) may or too-rapid (agitation, trembling, tension).
conditions, headache, hypoactivity, gravis. Children <6 decrease withdrawal Musculoskeletal spasm: Record
treatment of constipation, nausea, months of age. concentration. may result in onset, type, location, duration of
seizures (IV route blurred vision. CYP3A4 inhibitors pronounced pain. Check for immobility,
used for termination (e.g., itraconazole, restlessness, stiffness, swelling.
of status Rare: Paradoxical CNS Cautions: Pts ketoconazole) may irritability, INTERVENTION/EVALUATION
epilepticus). reactions (hyperactivity receiving other CNS increase insomnia, hand Monitor heart rate, respiratory rate,
/nervousness in children, depressants or concentration. tremor, B/P, mental status. Assess
excitement/restlessness in psychoactive agents, abdominal/muscle children, elderly for paradoxical
Gel: Control of elderly/debilitated) depression, history of cramps, reaction, particularly during early
increased seizure generally noted during first drug and alcohol abuse, HERBAL: Gotu diaphoresis, therapy. Evaluate for therapeutic
activity in 2 wks of therapy, renal/hepatic impairment, kola, kava kava, St. vomiting, response (decrease in
refractory epilepsy particularly in presence of respiratory disease, John’s wort, valerian seizures. intensity/frequency of
in pts on stable uncontrolled pain. impaired gag reflex, may increase CNS seizures; calm facial expression,
regimens. concurrent use of depression. St. decreased restlessness; decreased
strong CYP3A4 inhibitors John’s Abrupt intensity of skeletal muscle pain).
or inducers. withdrawal in Therapeutic serum level: 0.5–2
OFF-LABEL: wort may decrease pts with epilepsy mcg/ml; toxic serum level: greater
Treatment of panic concentration/effects. may produce than 3 mcg/ml.
disorder. Short-term ** increase
treatment of Seizures: Review history in
spasticity in of seizure disorder (length, FOOD: Grapefruit frequency/severity PATIENT/FAMILY TEACHING
children with intensity, frequency, products may of seizures. •Avoid alcohol. •Limit caffeine.
cerebral palsy. duration, LOC). Observe increase Overdose results •May cause drowsiness, avoid
Sedation for frequently for recurrence concentration/effects. in drowsiness, tasks that require alertness, motor
mechanically of seizure activity. Initiate confusion, skills until response to drug is
vented pts in ICU seizure precautions. diminished established. •May be habit
LAB VALUES: reflexes, CNS forming. •Avoid abrupt
None signifcant. depression, discontinuation after prolonged
Therapeutic serum coma. Antidote: use.
level: 0.5–2 mcg/ml; Flumazenil
toxic serum level:
greater than
3mcg/ml.
Trade Name Group Indication Side effect Precaution Interaction Adverse effects/ toxic Nursing care/ patient education
(Generic Name) reactions
Rivotril Anxiolytics / Adjunct in treatment of Frequent: Mild, Contraindications: DRUG: Abrupt withdrawal BASELINE ASSESSMENT
(Clonazepam) Anticonvulsants Lennox-Gastaut transient Narrow-angle Alcohol, other may result in Review history of seizure disorder
syndrome (petit mal drowsiness; ataxia, glaucoma, CNS pronounced (frequency, duration, intensity, level of
variant epilepsy); behavioral severe hepatic depressants may restlessness, irritability, consciousness [LOC]). For panic attack,
akinetic, myoclonic disturbances disease, pregnancy. increase CNS insomnia, hand assess
seizures; absence (aggression, depressant tremors, motor responses (agitation, trembling,
seizures (petit mal). irritability, effect. CYP3A4 abdominal/muscle tension), autonomic responses
Treatment of panic agitation), esp. in Cautions: inhibitors (e.g., cramps, diaphoresis, (cold/clammy
disorder. children. Renal/hepatic azole vomiting, status hands, diaphoresis).
impairment, impaired antifungals) epilepticus. Overdose
gag reflex, chronic may increase results in drowsiness,
OFF-LABEL: Restless Occasional: respiratory disease, concentration, confusion, diminished NTERVENTION/EVALUATION
legs syndrome, Dizziness, ataxia, elderly, debilitated toxicity. reflexes, Observe for excess sedation, respiratory
neuralgia, multifocal tic URI, fatigue. pts, depression, pts at coma. Antidote: depression, suicidal ideation. Assess
disorder, suicidal risk, or drug Flumazenil children, elderly for paradoxical reaction,
parkinsonian dysarthria, dependence. particularly during early therapy.
bipolar disorder, Rare: Impaired Initiate seizure precautions, observe
adjunct therapy for memory, dysarthria, frequently for recurrence of seizure
schizophrenia, nervousness, activity. Assist with ambulation if
sinusitis, rhinitis, drowsiness,
stomatitis, essential constipation, ataxia occur. For pts on long-term therapy,
tremor. allergic reaction. obtain LFT, renal function tests, blood
counts should be performed periodically.
Evaluate for therapeutic response:
decreased intensity and frequency of
seizures or, if used in panic
attack, calm facial expression, decreased
restlessness

PATIENT/FAMILY TEACHING
• Avoid tasks that require alertness, motor
skills until response to drug is
established. • Do not abruptly discontinue
medication after long-term therapy.
• Strict maintenance of drug therapy is
essential for seizure control. • Avoid
alcohol. • Report depression, thoughts
of suicide/self-harm, excessive drowsiness,
GI symptoms, worsening or loss of seizure
control
Trade Name Group Indication Side effect Precaution Interaction Adverse effects/ Nursing care/ patient education
(Generic Name) toxic reactions
Diazepam Anxiolytics, (Intravenous) Premedication Sedation, Precautions: Patient w/ - Isoniazid inhibits Somnolence, Nursing Consideration
Anticonvulsants, before anaesthesia, Sedation drowsiness, open angle glaucoma, metabolism. ataxia, confusion, - Assess baseline vital signs.
Hypnotics & in minor surgical and ataxia, muscle chronic pulmonary - Rifampicin increases dysarthria, little or - Assess blood pressure, pulse and
Sedatives medical procedures weakness, fatigue, insufficiency, muscle metabolism. no resp respiration if IV administration.
confusion, weakness, organic brain - Vomeprazole and depression, - Provide frequent sips of water for
depression, changes particularly esomeprazole may hypotension, dry mouth.
(Oral) headache, vertigo, arteriosclerosis, inhibit metabolism. muscular - Provide fluids and fibre for
Severe anxiety, muscle amnesia, personality disorder, - Ritonavir increases weakness, deep constipation.
spasms, adjunct in seizures, paradoxical phobia or on obsessional plasma levels - avoid coma, severe - Evaluate therapeutic response,
alcohol withdrawal reactions (e.g. state, chronic psychosis. concomitant use. depression, mental state and physical
syndrome, premedication anxiety, Use in patient w/ - Alters plasma levels diminished dependency after long-term use.
before anaesthesia, sedation hallucinations, depression or anxiety of phenytoin. reflexes.
in minor surgical and insomnia, associated w/ depression - Raises plasma level Management: Patient education
medical procedures, psychoses, sleep esp those w/ suicidal or of zotepine. Symptomatic and • Do not drink alcohol. • This
insomnia associated with disturbances), aggressive behaviour. - May antagonise supportive medication may impair your
anxiety visual History of drug and levodopa. treatment. Empty thinking or reactions. Be careful if
disturbances, alcohol addiction. Avoid stomach by you drive or do anything that
tremor, slurred abrupt withdrawal. Renal vomiting or gastric requires you to be alert. • Do not
(Parental) speech or and hepatic impairment. lavage. Activated use for everyday stress.s New
Severe anxiety, muscle dysarthria, Elderly and debilitated charcoal may help Roman
spasms, alcohol withdrawal paradoxical patient. Pregnancy and reduce absorption.
syndrome, seizures excitation, resp lactation. Flumazenil may
depression, be used for the
(Rectal)
hypotension, Contraindication: complete or partial
Adjunct in seizures, muscle changes in libido Patient w/ acute angle reversal of the
spasms, severe anxiety, and salivation, GI closure glaucoma, pre- sedative effects
premedication before disturbances, existing CNS depression, but there is a risk
anaesthesia, sedation in urinary retention coma, severe or acute of seizure esp in
minor surgical and medical or incontinence; resp insufficiency, sleep long-term
procedures pain and apnoea syndrome, benzodiazepine
thrombophlebitis myasthenia gravis, users and in cyclic
(IV). Rarely, severe hepatic antidepressant
hypersensitivity, impairment. Childn <6 overdose.
blood disorders, mth (oral).
jaundice,
increased liver
enzyme values.
Endocrine system
Trade Name Group Indication Side effect Precaution Interaction Adverse effects/ Nursing care/ patient education
(Generic Name) toxic reactions
Diamicron Antidiabetic Recommended in Hypoglycaemia: Contraindications: Drug: May increase Symptoms: Nursing consideration
(Gliclazide) Agents adults for non- Headache, intense Hypersensitivity to hypoglycaemic effect Hypoglycaemia monitor fasting and postprandial
insulin-dependent hunger, nausea, gliclazide or to any of w/ phenylbutazone. w/ or w/o coma, blood glucose and urinary glucose
diabetes (type 2), in vomiting, lassitude, sleep the excipients of Potentiation of blood convulsions or frequently. Monitor periodically
Belongs to the association with disorders, agitation, Diamicron, other glucose lowering other neurological during long-term therapy: Liver
class of dietary measures and aggression, poor sulphonylureas or effect w/ other disorders. function tests, serum osmolarity,
sulfonylureas with physical concentration, reduced sulphonamides, Type 1 antidiabetics (e.g. Management: serum sodium, and CBC with
exercise, when these awareness and slowed diabetes. Diabetic acarbose, insulins, Carbohydrate differential. Monitor for
measures alone are reactions, depression, precoma and coma, metformin), β- intake, dosage hypoglycemia especially with
not sufficient to confusion, visual and diabetic keto-acidosis. blockers, ACE adjustment and/or concurrent drugs which enhance
obtain normal blood speech disorders, Breastfeeding mother inhibitors, H2- change of diet hypoglycemic effects.
glucose levels (levels aphasia, tremor, paresis, receptor antagonists, may be helpful.
of sugar in the blood sensory disorders, Precaution: MAOIs, Admin rapid IV Patient Education
dizziness, feeling of Hypoglycaemia, Renal sulfonamides, inj of concentrated • Take a missed dose as soon as
powerlessness, loss of and Hepatic clarithromycin and glucose soln for possible unless it is almost time for
self-control, delirium, Insufficiency, Poor NSAIDs. hypoglycaemic next dose; NEVER take two doses
convulsions, shallow Blood Glucose Control Chlorpromazine, coma. at the same time. • Avoid drinking
respiration, bradycardia, glucocorticoids, alcohol or using OTC drugs
drowsiness and loss of ritodrine, salbutamol without informing physician. • Use
consciousness, possibly and terbutaline may sunscreen and avoid sunlamps. •
resulting in coma and cause increases in Learn about adverse reactions and
lethal outcome blood glucose levels. drug interactions. • Do not breast
May diminish feed while taking this drug.
hypoglycaemic effect
Adrenergic counter- w/ danazol. May
regulation: increase
Sweating, clammy skin, anticoagulant effect
anxiety, tachycardia, of warfarin.
hypertension, Potentially
palpitations, angina Fatal: Increased
pectoris and cardiac hypoglycaemic effect
arrhythmia w/ miconazole.

Food: May cause


RARE: disulfiram-like
Skin and Subcutaneous reactions w/ alcohol.
Tissue Disorders, Blood
and Lymphatic System
Disorders, Hepatobiliary
Disorders, Eye Disorders
Trade Name Group Indication Side effect Precaution Interaction Adverse effects/ Nursing care/ patient education
(Generic Name) toxic reactions
Glucophage Oral Management of Occasional: GI function; abnormal DRUG: Furosemide may Lactic acidosis BASELINE ASSESSMENT
(Metformin) Antidiabetic: type 2 diabetes disturbances creatinine clearance increase concentration. occurs rarely (0.03 Assess baseline glucose, Hgb A1c, CBC,
Biguanides mellitus as (diarrhea, nausea, from any cause Cationic medications (e.g., cases/1,000 pts) but renal function tests.
monotherapy or vomiting, abdominal including MI, acute digoxin, morphine, quinine, is a serious and often
concomitantly with bloating, flatulence, HF, ranitidine, vancomycin) may fatal (50%) INTERVENTION/EVALUATION
oral anorexia) that are septicemia, or increase complication. Lactic Monitor fasting serum glucose, Hgb A1c,
sulfonylurea or transient and resolve shock; acute or concentration/effects. acidosis is renal function, CBC. Monitor folic acid,
insulin. spontaneously during chronic metabolic Contrast agents may characterized by renal function tests for evidence of early
therapy. acidosis, use within increase risk of metformin- increase in lactic acidosis. If pt is on concurrent
OFF-LABEL: 48 hrs of IV induced lactic acidosis, acute blood lactate levels oral sulfonylureas, assess for
Polycystic ovarian Rare: Unpleasant contrast dye. renal failure (dis (greater than hypoglycemia (cool/wet skin, tremors,
syndrome, /metallic taste that Cautions: HF, continue metformin 24–48 5mmol/L), decrease dizziness anxiety, headache, tachycardia,
gestational resolves impaired hrs prior to and up to 72 hrs in blood pH, electro numbness in mouth, hunger, diplopia).
diabetes mellitus. spontaneously during hepatic function, after contrast exposure). lyte disturbances. Be alert to conditions that alter glucose re
Prevention of type therapy excessive Symptoms include quirements: fever, increased activity,
2 diabetes acute/chronic HERBAL: Garlic may cause unexplained stress, surgical procedure.
alcohol intake, hypoglycemia. hyperventilation,
elderly. myalgia, malaise, PATIENT/FAMILY TEACHING
LAB VALUES: May alter drowsiness. May • Discontinue metformin, report
cholesterol, LDL, advance to inmediately if evidence of lactic acidosis
triglycerides, HDL. cardiovascular appears (unexplained hyperventilation,
collapse (shock), muscle aches, extreme fatigue, unusual
acute drowsiness). • Prescribed diet is
HF, acute MI, principal part of treatment; do not skip,
prerenal azotemia. delay meals. • Diabetes mellitus re
quires lifelong control. • Avoid alcohol. •
Report persistent headache, nausea,
vomiting, diarrhea or if skin rash,
unusual bruising/bleeding, change
in color of urine or stool occurs.
Trade Name Group Indication Side effect Precaution Interaction Adverse effects/ toxic Nursing care/ patient education
(Generic Name) reactions
Actrapid HM Insulin Treatment of Hypoglycemia, Contraindication: The following substances hypoglycaemia may BASELINE ASSESSMENT
(insulin) preparation diabetes refraction Hypersensitivity to human may reduce the patient's develop over sequential Assess baseline glucose, Hgb A1c,
mellitus. anomalies, insulin or any of the excipients insulin requirement: Oral stages if too high doses CBC, renal function tests.
oedema, inj of Actrapid HM/Penfill. antidiabetic products, relative to the patient's
site reactions monoamine oxidase requirements are INTERVENTION/EVALUATION
(pain, redness, Precaution: inhibitors (MAOI), administered: Mild Monitor fasting serum glucose,
hives, Hyperglycemia: Inadequate nonselective β-blocking hypoglycaemic episodes Hgb A1c, renal function, CBC.
inflammation, dosage or discontinuation of agents, angiotensin can be treated by oral Monitor folic acid, renal function
bruising, treatment, especially in type 1 converting enzyme (ACE) administration of glucose tests for evidence of early
swelling & diabetes, may lead to inhibitors, anabolic steroids or sugary products. It is lactic acidosis. If pt is on
itching at the hyperglycaemia. In type 1 and sulphonamides. therefore recommended concurrent oral sulfonylureas,
inj. site). diabetes, untreated The following substances that the diabetic patient assess for hypoglycemia (cool/wet
hyperglycaemic events may increase the patient's always carry some sugar skin, tremors, dizziness anxiety,
eventually lead to diabetic insulin requirement: Oral containing products. headache, tachycardia, numbness
ketoacidosis, which is contraceptives, thiazides, Severe hypoglycaemic in mouth, hunger, diplopia). Be
potentially lethal. glucocorticoids, thyroid episodes, where the alert to conditions that alter
Hypoglycaemia: Occur if the hormones, patient has become glucose requirements: fever,
insulin dose is too high in sympathomimetics, growth unconscious, can be increased activity, stress, surgical
relation to the insulin hormone and danazol. treated with glucagon procedure.
requirement. Beta-blocking agents may (0.5-1 mg) given IM or
Omission of a meal or mask the symptoms of SC by a trained person, or PATIENT/FAMILY TEACHING
unplanned, strenuous physical hypoglycaemia and delay with glucose given IV by • Prescribed diet is principal part
exercise may lead to recovery from a healthcare professional. of treatment; do not skip, delay
hypoglycaemia. hypoglycemia. Glucose must also be meals. • Avoid alcohol. • Report
Continuous rotation of the Octreotide/lanreotide may given IV if the patient persistent headache, nausea,
injection site within a given area either increase or decrease dose not respond to vomiting, diarrhea or if skin rash,
may help to reduce or prevent the insulin requirement. glucagon within 10-15 unusual bruising/bleeding, change
these reactions. On rare Alcohol may intensify or min. in color of urine or stool occurs.
occasions, injection site reduce the hypoglycaemic Upon regaining
reactions may require effect of insulin. consciousness,
discontinuation of Actrapid Incompatibilities: Insulin administration of oral
HM/Penfill. products should only be carbohydrate is
added to compounds with recommended for the
Insulin preparations which have which it is known to be patient in order to prevent
been frozen must not be used. compatible. Medicinal relapse.
Insulin solutions should not be products added to the insulin
used if they do not appear clear solution may cause
and colourless. degradation of the insulin
eg, if the medicinal products
contain thiols or sulphites.
Actrapid HM/Penfill should not
be used in insulin pumps for
continuous SC insulin infusion.

Trade Name Group Indication Side effect Precaution Interaction Adverse effects/ toxic reactions Nursing care/
(Generic Name) patient education
Protaphane HM Insulin Treatment Hypoglycaemia, Contraindication: The Following Substances may Reduce the hypoglycemia may develop over sequential Patient education
(Neutral preparation of diabetes oedema, Hypersensitive, Patient's Insulin Requirement: Oral stages if too high doses relative to the •
Protamine mellitus. refraction Hypoglycemia antidiabetic products, monoamine oxidase patient's requirement are administered. Mild Store medicines at
Hagedorn anomalies, inj inhibitors (MAOIs), nonselective β- hypoglycaemic episodes can be treated by room temperature,
) site reactions. blocking agents, angiotensin converting oral administration of glucose or sugary away from heat
Caution: enzyme (ACE) inhibitors, salicylates, products. It is therefore recommended that and direct light. •
Inadequate dosage anabolic steroids and sulphonamides. the diabetic patient constantly carries some In case you miss a
or discontinuation The Following Substances may Increase sugar lumps, sweets, biscuits or sugary fruit dose, use it as
of treatment, the Patient's Insulin Requirement: Oral juice; Severe hypoglycaemic episodes, where soon as you
especially in type contraceptives, thiazides, glucocorticoids, the patient has become unconscious, can be notice. If it is
1 diabetes, may thyroid hormones, sympathomimetics, treated with glucagon (0.5-1 mg) (GlucaGen close to the time
lead to growth hormone and danazol. β-blocking Hypokit) given IM or SC by a person who of your next
hyperglycaemia. agents may mask the symptoms of has received appropriate instruction, or
hypoglycaemia and delay recovery from glucose given IV by a medical professional.
hypoglycaemia. Octreotide/lanreotide may Glucose must also be given IV if the patient
either increase or decrease the insulin does not respond to glucagon within 10-15
requirement. Alcohol may intensify or min. Upon regaining consciousness,
reduce the hypoglycaemic effect of insulin. administration of oral carbohydrate is
Incompatibilities: Insulin suspensions recommended for the patient in order to
should not be added to infusion fluids. prevent relapse.

Trade Name Group Indication Side effect Precaution Interaction Adverse effects/ toxic reactions Nursing care/
(Generic Name) patient education
Mixtard 30 Hm Insulin Treatment Hypoglycaemia, Contraindication: The Following Substances may Reduce the hypoglycemia may develop over Patient education
(insulin) preparation of DMs. refraction Hypersensitivity Patient's Insulin Requirement: Oral sequential stages if too high doses Store medicines at
antidiabetic products, monoamine oxidase relative to the patient's requirement are room temperature,
anomalies, oedema
Precaution: inhibitors (MAOIs), nonselective β-blocking administered. Mild hypoglycaemic away from heat and
& inj site reactions
Hyperglycemia and agents, angiotensin converting enzyme episodes can be treated by oral direct light. • In
(pain, redness,
hypoglycemia (ACE) inhibitors, salicylates, anabolic administration of glucose or sugary case you miss a
hives,inflammation,
steroids and sulphonamides. The Following products. It is therefore recommended dose, use it as soon
bruising, swelling &
Substances may Increase the Patient's that the diabetic patient constantly as you notice. If it
itching at the inj
Insulin Requirement: Oral contraceptives, carries some sugar lumps, sweets, is close to the time
site). Acute painful
thiazides, glucocorticoids, thyroid hormones, biscuits or sugary fruit juice; Severe of your next
neuropathy &
sympathomimetics, growth hormone and hypoglycaemic episodes, where the
temporary
danazol. β-blocking agents may mask the patient has become unconscious, can
worsening of
symptoms of hypoglycaemia and delay be treated with glucagon (0.5-1 mg)
diabetic retinopathy.
recovery from hypoglycaemia. given IM or SC by a person who has
Headache, Nausea,
Drowsiness, Octreotide/lanreotide may either increase or received appropriate instruction, or

Weakness,Dizziness decrease the insulin requirement. Alcohol glucose given IV by a medical


may intensify or reduce the hypoglycaemic professional. Glucose must also be
effect of insulin. Incompatibilities: Insulin given IV if the patient does not respond
suspensions should not be added to infusion to glucagon within 10-15 min. Upon
fluids. regaining consciousness

Trade Name Group Indication Side effect Precaution Interaction Adverse effects/ Nursing care/ patient education
(Generic Name) toxic reactions
prednisone Supportive Care Substitution therapy in Frequent: Insomnia, Contraindications: DRUG: Hepatic Long-term BASELINE ASSESSMENT
Therapy / deficiency states: Acute or heartburn, nervousness, Acute superfcial enzyme inducers therapy: Muscle Obtain baselines for height, weight,
Corticosteroid chronic adrenal insuffciency, abdominal distention, herpes simplex (e.g., wasting (esp. in B/P, serum glucose, electrolytes.
Hormones / Eye congenital adrenal diaphoresis, acne, mood keratitis, systemic phenobarbital, arms, legs), Check results of initial tests (TB
Corticosteroids / hyperplasia, adrenal swings, increased fungal infections, phenytoin, osteoporosis, skin test, X-rays, EKG). Never give
Antiasthmatic & insuffciency secondary to appetite, facial flushing, varicella, rifampin) may spontaneous live virus vaccine (e.g., smallpox).
COPD pituitary insuffciency. delayed wound healing, administration of live decrease effects. fractures,
Preparations / Nonendocrine disorders: increased susceptibility or attenuated virus Live virus amenorrhea, INTERVENTION/EVALUATION
Topical Arthritis, rheumatic carditis; to infection, diarrhea, vaccines. vaccines may cataracts, Monitor B/P, serum electrolytes,
Corticosteroids allergic, collagen, intestinal constipation. increase vaccine glaucoma, peptic glucose, results of bone mineral
tract, multiple sclerosis Cautions: side ulcer, HF. Abrupt density test, height, weight in
exacerbations; liver, ocular, Occasional: Headache, Hyperthyroidism, effects, potentiate withdrawal children. Be alert to infection (sore
renal, skin diseases; edema, change in skin cirrhosis, ocular virus replication, following long- throat, fever, vague symptoms);
bronchial asthma, cerebral color, frequent urination. herpes simplex, decrease pt’s term therapy: assess oral cavity daily for signs of
edema, malignancies. respiratory antibody response Anorexia, nausea, candida infection.
Rare: Tachycardia, tuberculosis, to vaccine. fever, headache,
OFFLABEL: Prevention of allergic reaction (rash, untreated systemic rebound PATIENT/FAMILY TEACHING
postherpetic neuralgia, relief urticaria), psychological infections, inflammation, • Report fever, sore throat, muscle
of acute pain in pts with changes, hallucinations, renal/hepatic fatigue, aches, sudden weight gain,
herpes depression. impairment; following weakness, swelling, loss of appetite, or
zoster, autoimmune acute MI, lethargy, dizziness, fatigue. •Avoid alcohol, minimize
hepatitis. diabetes, cataracts, orthostatic use of caffeine. •Maintain
glaucoma, seizures, hypotension. fastidious oral hygiene. • Do not
peptic ulcer disease, Sudden abruptly discontinue without
osteoporosis, discontinuance physician’s approval. • Avoid
myasthenia gravis, may exposure to chickenpox, measles.
hypertension, HF, be fatal.
ulcerative colitis,
thromboembolic
disorders, elderly.
Trade Name Group Indication Side effect Precaution Interaction Adverse effects/ Nursing care/ patient education
(Generic Name) toxic reactions
(Hydrocortisone) Supportive Care Injection: Soft tissue Frequent: Insomnia, Contraindications: DRUG: May Long-term INTERVENTION/EVALUATION
Therapy / inflammation heartburn, anxiety Fungal, tuberculosis, viral decrease effects of therapy: Assess for edema. Be alert to
Corticosteroid Intra-articular: Joint abdominal distention, skin lesions; serious diuretics, insulin, Hypocalcemia, infection (reduced immune
Hormones / Eye inflammations diaphoresis, acne, mood infections, oral hypokalemia, response): sore throat, fever, vague
Corticosteroids Intravenous: swings, increased appetite, IM administration in hypoglycemics, muscle wasting symptoms. Monitor daily pattern of
/ Antiasthmatic Supplement in facial idiopathic potassium (esp. arms, bowel activity, stool consistency.
& COPD adrenal insufficiency flushing, delayed wound thrombocytopenia supplements. legs), Monitor electrolytes, B/P, weight,
Preparations / during minor surgery healing, increased purpura. Hepatic enzyme osteoporosis, serum glucose. Watch for
Topical under general susceptibility to infection, inducers may spontaneous hypocalcemia (muscle twitching,
Corticosteroids anaesthesia, moderate diarrhea or constipation. Cautions: Thyroid decrease effects. fractures, cramps), hypokalemia (weakness,
or major surgery, dysfunction, cirrhosis, Live virus amenorrhea, paresthesia [esp. lower
Acute adrenocortical Occasional: Headache, hypertension, vaccines may cataracts, extremities], nausea/vomiting,
insufficiency edema, change in skin color, osteoporosis, decrease pt’s glaucoma, peptic irritability, EKG changes). Assess
Oral: Replacement frequent urination. Topical: thromboembolic antibody response ulcer, HF. Abrupt emotional status, ability to sleep.
therapy in Pruritus, tendencies or to vaccine, withdrawal after
adrenocortical redness, irritation. thrombophlebitis, HF, increase long-term therapy: PATIENT/FAMILY TEACHING
insufficiency seizure disorders, vaccine side Nausea, fever, • Report fever, sore throat, muscle
Topical/Cutaneous: Rare: Tachycardia, allergic diabetes, respiratory effects, potentiate headache, sudden aches, sudden weight gain,
Corticosteroid- reaction (rash, hives), tuberculosis, untreated virus replication. severe joint pain, swelling, visual disturbances,
responsive psychological changes, systemic infections, rebound behavioral changes. • Do not take
dermatoses hallucinations, depression. renal/hepatic impairment, inflammation, aspirin or any other medication
Topical: Allergic contact acute MI, fatigue, weakness, without consulting physician. •
dermatitis, purpura. myasthenia gravis, lethargy, Limit caffeine, avoid alcohol. •
Systemic: Absorption more glaucoma, cataracts, dizziness, Inform dentist, other physicians of
likely with use of occlusive increased intraocular orthostatic cortisone therapy now or within
dressings or extensive pressure, elderly hypotension. past 12 mos. • Caution against
application in young overusing joints injected for
children. symptomatic relief. • Topical:
Apply after shower or bath
for best absorption. • Do not cover
or use occlusive dressings unless
ordered by physician; do not use
tight diapers, plastic pants,
coverings. • Avoid contact with
eyes.
Musculoskeletal system
Trade Name Group Indication Side effect Precaution Interaction Adverse effects/ toxic Nursing care/ patient education
(Generic Name) reactions
Zyloprim Hyperuricemia PO: Management of Occasional: Contraindications: DRUG: Loop or thiazide Pruritic BASELINE ASSESSMENT
(Allopurinol) & Gout primary or secondary PO: None known. diuretics may increase maculopapular rash, Obtain baseline serum chemistries, hepatic
Preparations gout (e.g., acute attack, Drowsiness, level/effect. May possibly function tests. Instruct pt to drink
nephropathy). Treatment unusual Cautions: increase effect of oral accompanied by minimum of 2,500–3,000 ml of fluid
of secondary hair loss. Renal/hepatic anticoagulants. May malaise, fever, chills, daily while taking medication.
hyperuricemia that may IV: Rash, impairment, pts increase concentration, joint pain, nausea,
occur during cancer nausea, taking diuretics, toxicity of azathioprine vomiting, should be INTERVENTION/EVALUATION
treatment. Management vomiting. mercaptopurine or mercaptopurine. considered a toxic Discontinue medication immediately if
of recurrent uric acid and azathioprine, other Amoxicillin, ampicillin reaction. Severe rash or other evidence of allergic reaction
calcium oxalate calculi. Rare: drugs causing may increase incidence hypersensitivity occurs. Monitor I&O (output should
Diarrhea, myelosuppression. Do of rash. reaction may follow be at least 2,000 ml/day). Assess serum
Injection: Management headache. not use in appearance of rash. chemistries, uric acid, hepatic function.
of elevated uric acid in asymptomatic LAB VALUES: May Bone marrow Assess urine for cloudiness, unusual
cancer treatment for hyperuricemia increase serum BUN, depression, color, odor. Gout: Assess for therapeutic
leukemia, lymphoma, or alkaline phosphatase, hepatotoxicity, response: relief of pain, stiffness, swelling;
solid tumor ALT, AST, creatinine peripheral neuritis, increased joint mobility; reduced
malignancies. acute renal failure joint tenderness; improved grip strength.
occur rarely.
OFF-LABEL: In
mouthwash following PATIENT/FAMILY TEACHING
fluorouracil therapy to • May take 1 wk or longer for full
prevent stomatitis therapeutic effect. • Maintain adequate
hydration; drink 2,500–3,000 ml of fluid
daily while taking medication. • Avoid
tasks that require alertness, motor skills
until response to drug is established. •
Avoid alcohol (may increase
uric acid).
Trade Name Group Indication Side effect Precaution Interaction Adverse effects/ Nursing care/ patient education
(Generic Name) toxic reactions
Colcrys Belongs to Prevention, Frequent: Nausea, Contraindications: DRUG: May increase Bone marrow BASELINE ASSESSMENT
(Colchicine) Hyperuricemia treatment of vomiting, Concomitant use of concentration of depression (aplastic Obtain baseline laboratory studies. Gout:
& Gout acute gouty abdominal a P-glycoprotein (e.g., statins and increase risk of anemia, Assess involved joints for pain, mobility,
Preparations arthritis. Used discomfort. cyclosporine) or rhabdomyolysis. Atazanavir, agranulocytosis, edema. Mediterranean fever: Assess
to reduce Occasional: strong CYP3A4 inhibitor clarithromycin, thrombocytopenia) abdominal pain, fever, chills, erythema,
frequency of Anorexia. Rare: (e.g., clarithromycin) in cyclosporine, diltiazem, may occur with swollen skin lesions.
recurrence of Hypersensitivity presence of renal or erythromycin, fluconazole, long-term therapy.
familial reaction, hepatic impairment. fosamprenavir, indinavir, Overdose initially INTERVENTION/EVALUATION
Mediterranean including itraconazole, ketoconazole, causes burning Discontinue medication immediately if GI
fever angioedema. Cautions: Hepatic nelfnavir, ranolazine, feeling in symptoms occur. Encourage high fluid
(FMF). impairment, elderly, ritonavir, saquinavir, skin/throat; severe intake (3,000 ml/day). Monitor I&O
debilitated, renal verapamil may increase diarrhea, abdominal (output should be at least 2,000 ml/day),
OFF-LABEL: impairment. colchicine concentration, pain. Second stage CBC, hepatic/renal function tests. Monitor
Treatment of Concomitant use of toxicity. manifests as fever, serum uric acid. Assess for therapeutic
biliary cyclosporine, diltiazem, seizures, delirium, response: relief of pain, stiffness,
cirrhosis, verapamil, fbrates, FOOD: Grapefruit renal impairment swelling; increased joint mobility; reduced
recurrent statins; may increase products may increase (hematuria, joint tenderness; improved grip strength.
pericarditis. risk of myopathy. concentration/toxicity. oliguria). Third
stage causes PATIENT/FAMILY TEACHING
LAB VALUES: May hair loss, • Drink 8–10 glasses (8 oz) of fluid
increase serum alkaline leukocytosis, daily while taking medication. • Report
phosphatase, AST. May stomatitis. skin rash, sore throat, fever, unusual
decrease platelet count. bruising/bleeding, weakness, fatigue,
numbness. • Stop medication as soon
as gout pain is relieved or at frst sign of
nausea, vomiting, diarrhea. • Avoid
grapefruit products.
Trade Name Group Indication Side effect Precaution Interaction Adverse effects/ Nursing care/ patient education
(Generic Name) toxic reactions
Ibuprofen NSAID, Treatment of fever, juvenile Occasional: Contraindications: History DRUG: May Overdose may BASELINE ASSESSMENT
non-opioid rheumatoid arthritis (JRA), Nausea, of hypersensitivity to aspirin, decrease effects of result in metabolic Assess onset, type, location,
analgesics, osteoarthritis, minor to moderate vomiting, NSAIDs. Treatment of antihypertensives, acidosis. Rare duration of pain, inflammation.
antipyretic pain, primary dysmenorrhea. dyspepsia, perioperative pain in diuretics. Aspirin, reactions with Inspect appearance of affected
Caldolor: Mild to moderate pain; dizziness, rash. coronary artery bypass other salicylates long-term use joints for immobility, deformities,
severe pain in combination with an graft (CABG) surgery. may increase risk include peptic skin condition. Assess temperature.
opioid analgesic; fever. NeoProfen: Rare: Diarrhea NeoProfen: Infants of GI side ulcer, GI bleeding,
Induces closure in clinically or constipation, with proven or suspected effects, bleeding. gastritis, severe INTERVENTION/EVALUATION
signifcant patent ductus arteriosus flatulence, untreated infection, elevated May increase hepatic reaction Monitor for evidence of nausea,
(PDA) in premature infants abdominal total bilirubin, congenital effects of (cholestasis, dyspepsia. Monitor CBC, renal
weighing between 500 and 1,500 g cramps or pain, heart disease in whom oral jaundice), function, LFT. Assess skin for rash.
who are no more than 32 wks pruritus, patency of the patent ductus anticoagulants. nephrotoxicity Observe for bleeding, bruising,
gestational age when usual medical increased B/P arteriosus is necessary for May increase (dysuria, occult blood loss. Evaluate
management is ineffective. satisfactory pulmonary or concentration, risk hematuria, for therapeutic response: relief of
systemic blood flow (e.g., of toxicity of proteinuria, pain, stiffness, swelling; increased
intravenous: Closure of patent pulmonary atresia), bleeding, lithium, nephrotic joint mobility; reduced joint
ductus arteriosus, Pain relief, Fever thrombocytopenia, methotrexate. syndrome), severe tenderness; improved grip strength.
Oral: Juvenile rheumatoid arthritis, coagulation defects, hypersensitivity Monitor for fever.
Fever, Mild to moderate pain, suspected necrotizing Increased risk of reaction
Osteoarthritis, Rheumatoid arthritis, enterocolitis, signifcant renal GI bleeding w/ (particularly in pts
Dysmenorrhoea impairment. warfarin, with systemic PATIENT/FAMILY TEACHING
corticosteroids, lupus • Avoid aspirin, alcohol during
Topical/Cutaneous: Pain and Cautions: Pts with fluid SSRIs and aspirin. erythematosus or therapy (increases risk of GI
inflammation associated with retention, HF, coagulation May reduce the other collagen bleeding). • If GI upset occurs, take
musculoskeletal and joint disorders disorders, concurrent use natriuretic effects diseases). with food, milk, antacids. • May
with aspirin, anticoagulants, of diuretics. NeoProfen: cause dizziness. • Report ringing in
OFF-LABEL: Treatment of gout, steroids; history of GI disease Reduced Hypoglycemia, ears, persistent stomach pain,
acute migraine headaches, migraine (e.g., bleeding, ulcers); antihypertensive hypocalcemia, respiratory diffculty, unusual
prophylaxis, cystic fbrosis, smoking; use of alcohol; effect of ACE respiratory failure, bruising/ bleeding, swelling of
ankylosing, spondylitis. elderly, debilitated, inhibitors and UTI, extremities, chest pain/palpitations.
hepatic/renal impairment; angiotensin II edema, atelectasis
asthma. receptor may occur.
antagonists. May Caldolor:
increase toxicity Abdominal pain,
of lithium and anemia, cough,
methotrexate. dizziness,
Increased dyspnea, edema,
nephrotoxicity w/ hypertension,
ciclosporin and nausea, vomiting
tacrolimus. have been
reported.

Infection (Antibiotics) (IV&PO


Trade Name Group Indication Side effect Precaution Interaction Adverse effects/ Nursing care/ patient education
(Generic Name) toxic reactions
Augmentin Anti- Treatment of susceptible Occasional: Contraindications: DRUG: Antibiotic- BASELINE ASSESSMENT
(amoxicillin and microbrial infections due to Diarrhea, Hypersensitivity to Allopurinol may associated colitis, Question for history of allergies, esp.
clavulanate) agent.Belongs streptococci, E. coli, E. loose any increase other penicillins, cephalosporins, renal impairment.
to Penicillins faecalis, P. mirabilis, stools, penicillins, history incidence of superinfections
Class beta-lactamase producing nausea, skin of cholestatic rash. Probenecid (abdominal INTERVENTION/EVALUATION
H. influenzae, Klebsiella rashes, jaundice or hepatic may increase cramps, severe Hold medication and promptly report rash, diarrhea
spp., M. catarrhalis, and urticaria. impairment with concentration, watery diarrhea, (fever, abdominal pain, mucus and blood in stool may
S. aureus (not methicillin- amoxicillin/ toxicity risk. fever) may result indicate antibiotic-associated colitis). Be alert for signs
resistant Staphylococcus Rare : clavulanate therapy. May decrease from altered of superinfection including fever, vomiting, diarrhea,
aureus Vomiting, Augmentin XR: Se effect of oral bacterial balance. black “hairy” tongue,
[MRSA]) including lower vaginitis, vere renal contraceptives. Severe ulceration or changes of oral mucosa, anal/genital
respiratory skin and skin abdominal impairment, hypersensitivity pruritus. Monitor renal/hepatic tests with prolonged
structure, UTIs, otitis discomfort, hemodialysis pt. LAB VALUES: reactions, therapy.
media, sinusitis. flatulence, May increase including
headache Cautions: History serum ALT, AST. anaphylaxis, acute PATIENT/FAMILY TEACHING
OFF-LABEL: Chronic of allergies, esp. May cause interstitial •Continue antibiotic for full length of treatment. •Space
antimicrobial suppression cephalosporins; positive Coombs’ nephritis, occur doses evenly. •Take with meals if GI upset
of prosthetic joint renal impairment. test. rarely occurs. •thoroughly crush or chew the chewable tablets
infection. before swallowing. •Notify physician if rash, diarrhea

Trade Name Group Indication Side effect Precaution Interaction Adverse effects/ toxic Nursing care/ patient education
(Generic reactions
Name)
Tazocin Penicillins Lower resp tract infections, Rash; Contraindications: DRUG: Concurrent use Antibiotic-associated BASELINE ASSESSMENT
(piperacillin/ Class UTI (complicated & diarrhoea Hypersensitivity to of aminoglycosides may colitis, other Question for history of allergies, esp. to
tazobactam uncomplicated), intra- including any cause mutual inactivation superinfections penicillins, cephalosporins.
injection abdominal infections, skin & soft/loose penicillin. (must give at least 1 hr (abdominal cramps,
) skin structure infections, stools, apart). May increase severe watery INTERVENTION/EVALUATION
bacterial septicaemia; nausea, Cautions: History of concentration, toxicity of diarrhea, fever) may Monitor daily pattern of bowel activity,
polymicrobic infections. In vomiting. allergies (esp. methotrexate. Probenecid result from altered stool consistency; mild GI effects may be
combination w/ an cephalosporins, other may increase bacterial balance in GI tolerable, but increasing severity may
aminoglycoside for bacterial drugs), renal concentration, risk of tract. Overdose, indicate onset of antibiotic-associated
infections in neutropenic impairment, toxicity. High-dose more often with renal colitis. Be alert for superinfection: fever,
adult & childn. Appendicitis preexisting seizure piperacillin may increase impairment, may vomiting, diarrhea, anal/genital pruritus,
complicated by rupture w/ disorder. risk of bleeding produce seizures, oral mucosal changes (ulceration, pain,
peritonitis &/or abscess with heparin, NSAIDs, neurologic reactions. erythema). Monitor I&O, urinalysis.
formation in childn 2-12 yr. platelet inhibitors, Monitor serum electrolytes, esp.
thrombolytic agents, potassium, renal function tests.
warfarin.

Trade Name Group Indication Side effect Precaution Interaction Adverse effects/ toxic Nursing care/ patient education
(Generic reactions
Name)
Zinacef class of Treatment of Frequent: Discomfort with IM contraindication DRUG: Probenecid Antibiotic-associated BASELINE ASSESSMENT
(Cefuroxime) second- susceptible administration, oral candidiasis s: History of may increase colitis, other Obtain CBC, renal function tests.
generation infections due to (thrush), mild diarrhea, mild hypersensitivity/a concentration. superinfections Question for history of allergies,
cephalosp group B abdominal cramping, vaginal naphylactic Antacids, H2-receptor (abdominal cramps, particularly cephalosporins, penicillins.
orins. streptococci, candidiasis. reaction to antagonists (e.g, severe watery diarrhea,
Used in pneumococci, cephalosporins. cimetidine, famotidine) fever) may result from INTERVENTION/EVALUATION
the staphylococci, H. Occasional: Nausea, serum may decrease altered bacterial Assess oral cavity for white patches on
systemic influenzae, E. coli, sickness–like reaction (fever, Cautions: Severe absorption. balance. mucous membranes, tongue (thrush).
treatment Enterobacter, joint pain; usually occurs after renal impairment, Nephrotoxicity Monitor daily pattern of bowel activity,
Klebsiella second course of therapy and LAB VALUES: May may occur, esp. in pts stool consistency. Mild GI effects may be
of including acute/ resolves after drug is history of increase serum BUN, with preexisting renal tolerable (increasing severity may
infections. chronic discontinued). penicillin allergy. creatinine, disease. Pts with indicate onset of antibiotic-associated
bronchitis, alkaline phosphatase, history of penicillin colitis). Monitor I&O, renal function
gonorrhea, Rare: Allergic reaction (rash, bilirubin, LDH, ALT, allergy are at increased tests for nephrotoxicity. Be alert for
impetigo, early pruritus, urticaria), AST. May cause risk for developing a superinfection: fever, vomiting, diarrhea,
Lyme disease, thrombophlebitis (pain, redness, positive direct/indirect severe hypersensitivity anal/ genital pruritus, oral mucosal
otitis media, swelling at injection site) Coombs’ test reaction (severe changes (ulceration, pain, erythema).
pharyngitis/tonsill pruritus, angioedema,
itis, sinusitis, bronchospasm
skin/skin anaphylaxis). PATIENT/FAMILY TEACHING
structure, UTI, • Discomfort may occur with IM
perioperative injection. • Doses should be evenly
prophylaxis. spaced. • Continue antibiotic therapy for
full length of treatment. • May cause GI
upset (may take with food, milk).
Trade Name Group Indication Side effect Precaution Interaction Adverse Nursing care/ patient education
(Generic Name) effects/ toxic
reactions
Ampicillin Anti- Adult: In PO, for Frequent: Contraindications: May reduce the efficacy of OC. Nausea, BASELINE ASSESSMENT
microbrial Susceptible infections, Diarrhea, rash Hypersensitivity to May alter INR while on vomiting and Question for history of allergies, esp.
agent UTI, Typhoid and (most common), any warfarin and phenindione. May diarrhoea. penicillins, cephalosporins; renal impairment.
Belongs paratyphoid fever and urticaria, pain at penicillins or reduce the efficacy of oral Management:
to Uncomplicated IM injection site, sulbactam. typhoid vaccines. May reduce Symptomatic INTERVENTION/EVALUATION
Penicillins gonorrhoea. In IV, for thrombophlebitis the excretion of methotrexate. and supportive Hold medication and promptly report rash
Class Intrapartum prophylaxis with IV Cautions: History Reduced excretion w/ treatment. May (although common with ampicillin, may
against group B administration, of allergies, esp. probenecid and sulfinpyrazone, be removed indicate hypersensitivity) or diarrhea (fever,
Streptoccocal infection oral or vaginal cephalosporins, resulting to increased risk of from the abdominal pain, mucus and
and Meningitis. In candidiasis. renal impairment, toxicity. Allopurinol increases circulation by blood in stool may indicate antibioticassociated
Intrapleural/IP/IA, for Occasional: infectious ampicillin-induced skin haemodialysis. colitis). Evaluate IV site for phlebitis. Check IM
Supplement in systemic Nausea, mononucleosis, reactions. Reduced absorption injection site for pain, induration. Monitor I&O,
therapy for treatment of vomiting, asthmatic pts. w/ chloroquine. Bacteriostatic urinalysis, renal function tests. Be alert for
susceptible infections. In headache, antibacterials (e.g. superinfection: fever, vomiting, diarrhea,
IM/IV, for Septicaemia malaise, urinary erythromycin, anal/genital pruritus, oral mucosal changes
and Susceptible retention. chloramphenicol, tetracycline) (ulceration, pain, erythema).
infections. may interfere w/ the
bactericidal action of PATIENT/FAMILY TEACHING
ampicillin. • Take antibiotic for full length oftreatment. •
Space doses evenly. •Discomfort may occur
with IM injection. • Report rash, diarrhea, or
other new symptoms
Trade Name Group Indication Side effect Precaution Interaction Adverse effects/ Nursing care/ patient education
(Generic toxic reactions
Name)
Cefazolin Cephalosporins Adult: In IV/IM, for Frequent: Discomfort Contraindications: DRUG: Probenecid may Antibiotic- BASELINE ASSESSMENT
(Ancef) class Susceptible infections, with IM administration, History of increase concentration associated colitis, Obtain CBC, renal function tests.
Acute uncomplicated UTI, oral candidiasis (thrush), hypersensitivity/ other Question for history of allergies,
Pneumonia and mild diarrhea, mild anaphylactic reaction May enhance the superinfections particularly cephalosporins,
Prophylaxis of surgical abdominal cramping, to cephalosporins. anticoagulant effect of (abdominal penicillins.
infections. vaginal candidiasis. vit K antagonists (e.g. cramps, severe
Cautions: Severe warfarin). May diminish watery diarrhea, INTERVENTION/EVALUATION
Treatment of susceptible Occasional: Nausea, renal impairment, the therapeutic effect of fever) may result Evaluate IM site for induration and
infections due to S. serum sickness-like history of penicillin Na picosulfate, BCG and from tenderness. Assess oral cavity for
aureus, S. epidermidis, reaction (fever, joint allergy, history typhoid vaccine. May white patches on
group A beta-hemolytic pain; usually occurs after of seizures. Renal decrease the protein altered bacterial mucous membranes, tongue
streptococci, S. second course of impairment. binding of fosphenytoin balance. (thrush). Monitor daily pattern of
pneumoniae, E. coli, P. therapy and resolves Pregnancy and and phenytoin. Nephrotoxicity bowel activity, stool consistency.
mirabilis, Klebsiella spp., after drug is lactation. Probenecid may decrease may occur, esp. Mild GI effects may be tolerable
H. influenzae including discontinued). renal tubular secretion of in pts with (increasing severity may indicate
biliary tract, bone and cefazolin, resulting in preexisting onset of antibiotic-associated
joint, genital, respiratory Rare: Allergic reaction increased and prolonged renal disease. Pts colitis). Monitor I&O, renal
tract, skin/skin structure (rash, pruritus, urticaria), blood levels. May with history of function tests for nephrotoxicity. Be
infections; endocarditis, thrombophlebitis increase the nephrotoxic penicillin allergy alert for superinfection: fever,
perioperative (pain, redness, swelling effects of are at increased vomiting, diarrhea, anal/genital
prophylaxis, septicemia. at injection site). aminoglycosides. risk for pruritus, oral mucosal changes
developing severe (ulceration, pain, erythema).
OFF-LABEL: Prophylaxis hypersensitivity
against infective reaction PATIENT/FAMILY TEACHING
endocarditis (severe pruritus, • Discomfort may occur with IM
angioedema, injection.
bronchospasm,
anaphylaxis).
Trade Name Group Indication Side effect Precaution Interaction Adverse effects/ Nursing care/
(Generic Name) toxic reactions patient education
Cloxacillin Anti-microbrial agent Staphylococcal Hypotension, confusion, fever, lethargy, Hypersensitivity to May diminish the effect Diarrhea PATIENT/FAMILY

Belongs to Penicillins infections seizure, pruritus, rash, urticaria, abdominal to cloxacillin and of BCG and typhoid Feel Like TEACHING
resistant to pain, black or hairy tongue, diarrhoea, other penicillins. vaccine. May increase Throwing Up • Discomfort may
Class
benzylpenicillin. flatulence, nausea, oral candidiasis, Patient w/ allergies risk of methotrexate Rash occur with IM
pseudomembranous colitis, stomatitis, esp β-lactam toxicity. May diminish Throwing Up injection.
vomiting, agranulocytosis, bone marrow allergy, asthma, the therapeutic effect of
depression, eosinophilia, granulocytopenia, history of seizure Na picosulfate. May
haemolytic anaemia, leucopenia, disorder. Renal decrease serum
neutropenia, thrombocytopenia, increased impairment. concentrations of
alkaline phosphatase, ALT and AST, Pregnancy an mycophenolate. May
hepatotoxicity, thrombophlebitis, prolong bleeding time w/
arthralgia, myalgia, myoclonus, hematuria, anticoagulants. Increased
interstitial nephritis, proteinuria, renal serum concentrations w/
insufficiency, renal tubular damage, probenecid. Tetracycline
bronchospasm, laryngeal oedema, may antagonise the
laryngospasm, sneezing, wheezing, bactericidal effect of
angioedema, allergic reaction, serum cloxacillin.
sickness-like reaction.
Potentially Fatal: Anaphylaxis.

You might also like